Usmle Step1 Q&A Mukhtar

Download as pdf or txt
Download as pdf or txt
You are on page 1of 147

246 questions of step 1

You have 246 questions in this exam.

1. Which of the following is/are true relative to the definition, diagnosis


and prevalence of hypertension.
1. A BP cuff that is too small (i.e., encircles less than 80% of the upper
arm) gives an erroneously low BP reading.
2. The upper normal BP value for a seven-year-old is 135/85.
3. African Americans experience a higher prevalence of hypertension than
Caucasians, but appear to suffer less severe end-organ damage.
4. The prevalence of hypertension rises with age.

A. Only choices 1, 2, and 3 are correct


B. Only choices 1 and 3 are correct
C. Only choices 2 and 4 are correct
D. Only 4 is correct
E. All choices are correct

Show answer
Correct Answer: D

2. Which of the following is/are true relative to the causes of high blood
pressure:
1. Renal parenchymal disease is the single most common cause of persistent
hypertension in the pre-adolescent population.
2. Environmental factors thought to raise BP include obesity, diabetes, high
salt intake, physical inactivity and immoderate alcohol consumption.
3. Renal artery stenosis, renal parenchymal disease, endocrine disorders,
pregnancy, and drugs, account for the majority of secondary hypertension.
4. A cause for hypertension can be identified in close to 95% of hypertensive
adults, if diligently searched for.

A. Only choices 1, 2, and 3 are correct


B. Only choices 1 and 3 are correct
C. Only choices 2 and 4 are correct
D. Only 4 is correct
E. All choices are correct

Show answer
Correct Answer: A

3. The work-up of a patient who is referred to you with established


hypertension should include:
1. A thorough drug history.
2. Physical examination to include assessment of the skin.
3. Urinalysis, serum electrolytes and creatinine.
4. Assessment of left ventricular heart mass.

A. Only choices 1, 2, and 3 are correct


B. Only choices 1 and 3 are correct
C. Only choices 2 and 4 are correct
D. Only 4 is correct
E. All choices are correct

Show answer
Correct Answer: E

4. The following is/are true regarding the treatment of essential


hypertension:
1. The elderly usually require and tolerate larger doses of diuretics and beta
blockers than younger adults.
2. Beta blockers should generally be avoided in those with reactive airway
disease (asthma).
3. Beta blockers and angiotensin-converting enzyme (ACE) inhibitors should be
avoided following myocardial infarction.
4. The slow reduction of medication (step-down therapy) can be attempted in
those with essential hypertension who have been under good control for one
year.

A. Only choices 1, 2, and 3 are correct


B. Only choices 1 and 3 are correct
C. Only choices 2 and 4 are correct
D. Only 4 is correct
E. All choices are correct

Show answer
Correct Answer: C

5. Which statements are true regarding the increase in glomerular filtration


rate (GFR) that occurs after birth?
1. Increased renal blood flow contributes to increased GFR in the neonate.
2. GFR increases immediately after birth, regardless of gestational age.
3. Redistribution of blood flow to the outer renal cortex increases GFR.
4. Renal vascular resistance increases while systemic vascular resistance
falls after birth.

A. Only choices 1, 2, and 3 are correct


B. Only choices 1 and 3 are correct
C. Only choices 2 and 4 are correct
D. Only 4 is correct
E. All choices are correct

Show answer
Correct Answer: B

6. Tubular reabsorption of _______________ increases with maturation.


1. Sodium
2. Bicarbonate
3. Potassium
4. Phosphate

A. Only choices 1, 2, and 3 are correct


B. Only choices 1 and 3 are correct
C. Only choices 2 and 4 are correct
D. Only 4 is correct
E. All choices are correct

Show answer

Correct Answer: A

7. A three month old infant born at full term weights 5 kg and has a length of
50 cm. The serum creatinine is 1.0. Which statements are true?
1. This infant has normal renal function.
2. Glomerulogenesis is complete.
3. Tubular maturation is complete.
4. This infant's glomerular filtration rate is approximately 22 mL/1.73
m2/min.

A. Only choices 1, 2, and 3 are correct


B. Only choices 1 and 3 are correct
C. Only choices 2 and 4 are correct
D. Only 4 is correct
E. All choices are correct

Show answer
Correct Answer: C

8. The following statements about congenital nephrotic syndrome of the Finnish


type are true.
1. It is caused by a mutation in the nephrin gene on chromosome 19.
2. Maternal alpha feto protein is increased in the second trimester.
3. Steroid therapy is useless.
4. Proteinuria is present prior to age three months.

A. Only choices 1, 2, and 3 are correct


B. Only choices 1 and 3 are correct
C. Only choices 2 and 4 are correct
D. Only 4 is correct
E. All choices are correct

Show answer
Correct Answer: E

9. Factors contributing to renal vein thrombosis include all of the following


EXCEPT:
A. Hemoconcentration
B. Increased antithrombin Ill levels
C. Increased platelet activation
D. High molecular weight fibrinogen

Show answer
Correct Answer: B

10. Findings in atheroembolic renal disease include:


A. Renal failure
B. Eosinophilia
C. Hypocomplementemia
D. Livedo reticularis
E. All of the above

Show answer
Correct Answer: E

11. The most likely diagnosis in a patient presenting with signs and symptoms
of small vessel vasculitis, antineutrophil cytoplasmic antibodies (ANCA) in
the blood and no evidence of asthma, eosinophilia or necrotizing granulomas is
which of the following:
A. Henoch-Schonlein purpura
B. Cryoglobulinemic vasculitis
C. Microscopic polyangiitis
D. Necrotizing granulomatosis (Wegener's)
E. Churg-Strauss syndrome

Show answer
Correct Answer: C

12. A 45-year-old physician has a long history of recurrent kidney stones.


Because of his busy schedule as Chief of Medicine he has undergone 11
extracorporeal shock wave lithotripsy (ESWL) treatments to "get rid of the
stones". He states that he can handle his stone problem quite well in this

manner and that he has seen no reason to have a metabolic work-up. He noticed
in recent months, however, that he has to urinate quite often, especially at
night, that he has exertional dyspnea and that he is more fatigued than usual.
All of the following statements apply to his situation, EXCEPT:
A. ESWL reduces stone activity.
B. ESWL can increase stone activity.
C. Repeated ESWL can cause hypertension and 2o heart failure
D. Repeated ESWL can cause renal insufficiency.
E. Despite a good response to ESWL, a metabolic evaluation is essential.

Show answer
Correct Answer: A

13. Which therapeutic measures correlate best with the prevention of Calcium
oxalate stones in patients with idiopathic hypercalciuria?
A. Extracorporeal Shock Wave Lithotripsy.
B. sustained alkalinization of the urine with bicarbonate.
C. a high sodium diet.
D. high dose vitamin C.
E. low sodium and normal protein intake, a high urine volume, thiazide
diuretic.

Show answer
Correct Answer: E

14. Which one of the following factors contribute to edema formation in


congestive heart failure?
A. Effective arterial blood volume is increased due to renal salt and water
retention.
B. Decreased effective arterial blood volume leads to increased aldosterone,
ADH, and sympathetic nerve activity.
C. Total peripheral resistance is decreased due to splanchnic vasodilatation.
D. Decreased actual blood volume leads to activation of baroreceptors.
E. The renin-angiotensin system is suppressed.

Show answer
Correct Answer: B

15. Which one of the following does not drive potassium into cells?
A. Insulin
B. Increased extracellular pH
C. Epinephrine
D. Increased extracellular sodium concentration
E. Aldosterone

Show answer
Correct Answer: D

16. The clinical manifestations of hyperkalemia include all of the following


except:
A. Predisposes to digitalis toxicity
B. Predisposes to ventricular fibrillation
C. EKG shows flattened P wave, peaked T wave, and widened QRS complex
D. Weakness
E. Predisposes to cardiac arrest

Show answer
Correct Answer: A

17. Which one of the following statements is false?


A. The main cause of hypokalemia associated with vomiting is not loss of K+ in
the vomitus.
B. Diuretics are one of the most common causes of hypokalemia.
C. Renal failure is a common cause of hyperkalemia.
D. Hyperkalemia in diabetes mellitus is at least partially due to inadequate
aldosterone formation.
E. Copious watery diarrhea is frequently associated with hyperkalemia.

Show answer
Correct Answer: E

18. An elderly woman develops a urinary tract infection, becomes confused, and
does not eat or drink for three days at her nursing home. She is found to have
a blood pressure of 70/50 mmHg (very low). Which one of the following

intravenous fluids would you recommend?


A. One-half normal saline
B. Normal saline
C. 5% dextrose and water
D. Hypertonic saline
E. One-quarter normal saline

Show answer
Correct Answer: B

19. Which one of the following statements about hypo- or hypernatremia is


true?
A. A patient with frank symptoms of hypo- or hypernatremia should have his/her
serum sodium concentrations rapidly (few hours) corrected back to normal
levels.
B. The brain fully compensates for hypenatremia within 2-4 hours by making
"idiogenic osmoles".
C. The symptoms of hyper- and hyponatremia are mainly due to central nervous
system dysfunction.
D. Hyponatremia due to SIADH is most often due to underlying kidney or liver
disease.
E. "Pseudohyponatremia" is as dangerous as true hyponatremia.

Show answer
Correct Answer: C

20. Which of the following is NOT commonly found in chronic urinary


obstruction?
A. Nocturia
B. No symptoms
C. Renal failure
D. Microhematuria with dysmorphic red cells
E. Polyuria

Show answer
Correct Answer: D

21. A 23-year-old sexually active woman presents with urinary frequency and
burning on micturition of 24 hours duration. Her urinalysis shows: Yellow,
hazy urine. pH 6. No glucose. Protein trace, blood trace. 20-50 WBC/HPF, 10-20
RBC/HPF. 0-1 squamous epithelial cells and no bacteria. You prescribe
trimethoprim/sulfamethoxazole tablets twice a day. The next day she feels much
better but her urine culture report reads:"Staphylococcus saprophyticus
100,000 cfu/ml." Which of the following statements is true?
A. She has a bacterial UTI.
B. The urine sample was contaminated and the results from it are unreliable.
C. The hematuria likely signifies the presence of a stone or tumor.
D. The Staphylococcus saprophyticus is likely a skin contaminant.
E. The urinalysis is a laboratory error.

Show answer
Correct Answer: A

22. A 58-year-old woman presents with hematuria and left flank pain. Renal
ultrasound shows marked left hydronephrosis. Serum creatinine and CBC are
normal. Which of the following is the most relevant diagnostic aid?
A. Renal biopsy
B. Renal arteriogram
C. Palpation for inguinal lymph nodes
D. Antinuclear antibody test
E. Pelvic examination and IVP

Show answer
Correct Answer: E

23. A 26 year-old diabetic woman is seen in the ER for sore throat. Rapid
strep test is positive for streptococcal pharyngitis and she was started on
ampicillin 500 mg four times a day. Three days later, she develops hematuria
associated with a low grade fever. On physical examination, she has a
maculopapular rash and a temperature of 101oF. Laboratory studies show: serum
creatinine 3.6 mg/dl, WBC 8,700 with 56% PMN, 25% lymphs, 3% monos and 15%
eosinophils. Urinalysis: pH 6.2, protein 2+, blood 3+, 65 RBCs/HPF, 20-30
WBCs/HPF, 3-4 WBC casts/HPF. Hansel's stain is positive for eosinophils. The
most likely diagnosis would be:
A. Diabetic nephropathy
B. IgA nephropathy
C. Acute interstitial nephritis

D. Acute pyelonephritis
E. Acute post-streptococcal glomerulonephritis

Show answer
Correct Answer: C

24. Analgesic nephropathy can be associated with all of the following except:
A. Small kidney size by renal ultrasound
B. Nephritic sediment with RBC casts
C. Increased risk of transitional cell carcinoma
D. Tubulointerstitial fibrosis on renal biopsy
E. Urinary tract obstruction due to papillary necrosis on IVP

Show answer
Correct Answer: B

25. Autosomal dominant polycystic kidney disease is associated with all of the
following except:
A. Hypertension is very common
B. Ultrasound is very useful in establishing the diagnosis
C. Renal failure progresses slowly
D. The majority of the patients (>80%) have cerebral aneurysms
E. Lipid soluble antibiotics should be used when the cysts become infected

Show answer
Correct Answer: D

26. A 10-month-old boy presents with status epilepticus. His intoxicated


mother was unable to provide a history. The child quit seizing after being
treated with diazepam. Exam revealed an obtunded child with tachypnea.
Screening labs showed Na 140 mEq/L, K 5.5 mEq/L, total CO2 6 mEq/L, Cl 104
mEq/L, BUN 15 mg/dl, creatinine 0.6 mg/dl, glucose 40 mg/dl, Ca 9.5 mg/dl, Mg
1.4 mg/dl, PO4 5 mg/dl, serum osmolality 350 mosm/L. The arterial blood gas
showed a pH of 7.0, PCO2 25 mmHg. The patient has a:
A. Metabolic acidosis
B. Respiratory acidosis
C. Metabolic alkalosis

D. Respiratory alkalosis

Show answer
Correct Answer: A

27. The anion gap in the infant above is:


A. 25
B. 30
C. 36
D. None of the above

Show answer
Correct Answer: B

28. What is his possible diagnosis:


A. Septic shock with lactic acidosis
B. Intoxication with methanol
C. Diabetic ketoacidosis
D. Bartter's syndrome

Show answer
Correct Answer: B

29. An infant presents to a pediatrician for recurrent vomiting and failure to


thrive. The child has been hospitalized by another primary care physician for
recurrent dehydration. However, data are not available. Weight and length are
below the fifth percentile. Exam shows a normal blood pressure with signs of
mild dehydration. Reflexes are markedly increased. The child is hypotonic.
Chemistry panel showed Na 140 mEq/L, K 2.2 mEq/L, CO2 50 mEq/L, Cl 78 mEq/L,
BUN 20 mg/dl, creatinine 1.0 mg/dI, Ca 9.5 mg/dl, Mg 1.7 mg/dl, PO4 2.8 mg/dl.
Arterial blood gas showed pH 7.54, pCO2 60 mmHg, urine chloride was 40 mEq/L
and urinary calcium excretion was elevated. What is the nature of the
acid-base disorder?
A. Respiratory acidosis
B. Metabolic alkalosis
C. Respiratory alkalosis
D. Metabolic acidosis

Show answer
Correct Answer: B

30. The most likely diagnosis in this infant is:


A. Gitleman's syndrome
B. Bartter's syndrome
C. Renal tubular acidosis
D. Contraction alkalosis

Show answer
Correct Answer: B

31. Which one of the following is clearly associated with autosomal recessive
polycystic kidney disease?
A. Very large cysts throughout the kidney parenchyma, including cortex and
medulla
B. Cerebral aneurysms
C. Gastrointestinal diverticulum
D. Congenital hepatic fibrosis

Show answer
Correct Answer: D

32. Which one of the following is true regarding urinary tract infections
(UTI) in children?
A. UTI are equally common in boys and girls.
B. UTI are more common in boys during the neonatal period .
C. UTI are always more common in girls throughout childhood.
D. UTI are almost never associated with vesicoureteral reflux in children.

Show answer
Correct Answer: B

33. Hemolytic uremic syndrome in children


A. Is usually non-diarrheal.
B. Is most often associated with colitis from enterohemorrhagic E. coli.
C. Is always benign.
D. Is best treated with plasma infusions.

Show answer
Correct Answer: B

34. Which one of the following regarding glomerular filtration rate (GFR) is
true:
A. Creatinine clearance is seldom used as a marker of GFR in clinical medicine
because it consistently under- estimates true GFR.
B. Two patients with identical serum creatinine concentration always have the
same GFR.
C. In advanced renal failure (GFR <15 ml/min), 24-hour urea clearance is
useful in conjunction with 24-hour creatinine clearance to estimate GFR.
D. For a given individual, an increase of serum creatinine from 8.0 mg/dl to
10.0 mg/dl represents a loss of greater number of nephrons than an increase of
serum creatinine from 1.0 mg/dl to 2.0 mg/dl.
E. Muscle mass is the predominant determinant of creatinine clearance.

Show answer
Correct Answer: C

35. A low blood urea nitrogen to serum creatinine ratio (<10:1) is seen in one
of the following conditions:
A. administration of corticosteroids for arthritis
B. catabolic state during trauma
C. advanced liver failure
D. high protein diet
E. volume depletion from gastrointestinal bleeding

Show answer
Correct Answer: C

36. Which of the following is most likely to be associated with high urinary

sodium concentration (>40 mEq/L):


A. low salt intake in a normal subject
B. severe congestive heart failure
C. diarrhea in an otherwise normal adult
D. acute tubular necrosis

Show answer
Correct Answer: D

37. In a patient with oliguria, which ONE of the following favors the
diagnosis of acute tubular necrosis rather than pre-renal azotemia:
A. ratio of urine osmolality to plasma osmolality = 2.0
B. ratio of urine creatinine concentration to plasma creatinine concentration
> 50
C. fractional excretion of sodium (FENa) > 2%
D. urinary sodium concentration <5 mEq/L

Show answer
Correct Answer: C

38. A patient reports that he has noticed a decrease in urine output for five
days. His BUN is 100 mg/dl and his serum creatinine is 7 mg/dl. Which of the
following is LEAST useful in establishing the etiology of his renal failure:
A. urine sodium concentration
B. ultrasound of the kidneys
C. intravenous pyelogram
D. urinalysis
E. history and physical

Show answer
Correct Answer: C

39. Which of the following is NOT an indication for urgent dialysis for a
patient with renal failure:
A. asterixis and drowsiness
B. pulmonary edema resistant to diuretics
C. pericarditis

D. serum creatinine of 12 mg/dL


E. serum potassium of 9.0 mEq/L

Show answer
Correct Answer: D

40. In the management of patients with chronic glomerulonephritis and chronic


renal failure with a serum creatinine of 2.0 mg/dl, which of the following is
true:
A. Angiotensin converting enzyme inhibitors are contraindicated because they
are toxic to the kidneys in renal failure.
B. Dosage of medications often require adjustment.
C. Nonsteroidal anti-inflammatory agents (arthritis medications) can be used
liberally because the chronically diseased kidneys are resistant to the toxic
effects of these agents.
D. Superimposed urinary tract obstruction is of no consequence because the
underlying kidney disease is glomerular in origin.
E. Severe sodium restriction is always necessary because the chronically ill
kidneys cannot excrete sodium.

Show answer
Correct Answer: B

41. The most common cause of end stage renal disease in the U.S.A. is:
A. hereditary nephritis
B. sickle cell nephropathy
C. post-infectious glomerulonephritis
D. renal cell carcinoma
E. diabetic nephropathy

Show answer
Correct Answer: E

42. Which of the following is most commonly encountered in a patient with


chronic renal failure and bone biopsy showing osteitis fibrosa cystica:
A. increased serum alkaline phosphatase level
B. markedly decreased serum phosphorus concentration

C. increased serum 1 ,25(OH)2D3 level


D. low parathormone level
E. increased serum calcium concentration

Show answer
Correct Answer: A

43. Which of the following is the most common complication of advanced


untreated renal failure:
A. erythrocytosis
B. platelet dysfunction favoring bleeding
C. increased peripheral nerve conduction
D. gastrointestinal tumors
E. hypotriglyceridemia

Show answer
Correct Answer: B

44. In the evaluation of proteinuria, which of the following is false:


A. Proteinuria as detected by the dipstick methods is quite insensitive, and
may miss albuminuria in early diabetic nephropathy.
B. The dipstick method may not detect Bence-Jones proteins in the urine.
C. A random urine sample for protein:creatinine ratio often suffices in
following proteinuria in a patient under treatment, as long as there is no
substantial change in muscle mass.
D. Similar to sodium, protein excretion in the urine is a good reflection of
dietary protein intake.

Show answer
Correct Answer: D

45. Based on the following data (BUN = 40 mg/dl, 24 urine volume = 2 liters,
serum creatinine = 2 mg/dL, urine creatinine = 40 mg/dL, serum sodium = 150
mEq/dL, urine sodium = 15 mEq/L) the fractional excretion of sodium (FENa) can
be calculated to be:
A. 0.2%
B. 0.5%

C. 5%
D. 20%
E. 50%

Show answer
Correct Answer: B

46. Which one of the following statements is correct:


A. Diabetic nephropathy and proteinuria rarely develop after 10 years duration
of diabetes.
B. Patients with type I diabetes and diabetic nephropathy do not have higher
risk of death than those without diabetic nephropathy.
C. All patients with diabetic nephropathy have type I DM (diabetes mellitus).
D. Diabetes mellitus is the leading cause of ESRD in the U.S.A.
E. If a patient survives 40 years of DM without developing nephropathy, he or
she is at extremely high risk of doing so in the future.

Show answer
Correct Answer: D

47. Which one of the following statements is not correct?


A. Hyperglycemia leads to the accumulation of advanced glycosylation end
products (AGE's) in tissues in patients with DM.
B. AGE's accumulation in tissues parallels the severity of renal disease.
C. Decreasing intraglomerular pressure preserves the structure and function of
the glomerulus in diabetic patients.
D. AGE's are responsible for end organ damage seen in diabetes.
E. Hypertension does not add any significant risk for developing renal disease
in diabetic patients.

Show answer
Correct Answer: E

48. Which one of the following is correct:


A. Advanced diabetic nephropathy means development of microalbuminuria.
B. Normal urine albumin excretion in normal people varies between 400-500
mg/24 hr.

C. Microalbuminuria in diabetic patients predicts the development of frank


proteinuria and ESRD.
D. Microalbuminuria in diabetic nephropathy patients doesn't become manifest
until serum creatinine is higher than 2.0 mg/dL.
E. Once frank proteinuria is established, almost all patients reach ESRD
within one year.

Show answer
Correct Answer: C

49. Which one of the following is not correct:


A. All patients with type I diabetes of greater than 5 years duration should
have an annual screen for microalbuminuria to identify this high risk
population.
B. 90-95% of patients with diabetic nephropathy have diabetic retinopathy.
C. The absence of retinopathy should make one suspect a cause of proteinuria
other than diabetic nephropathy.
D. Frank proteinuria after only 5 years duration of type I DM is unlikely to
be secondary to diabetic nephropathy.
E. If the urinary albumin excretion rate is greater than 30 mg/24 hr in a
single urine collection, this microalbuminuria does not need to be confirmed
with additional collections since other causes of microalbuminuria (e.g.,
hypertension, CHF) are rare in di

Show answer
Correct Answer: E

50. Regarding the management of diabetes mellitus, all of the following are
correct EXCEPT:
A. Dialysis therapy, in patients with diabetic nephropathy and ESRD is not
necessary until GFR is less than 5 ml/min.
B. Tight blood sugar control reduces the risk of developing diabetic
nephropathy.
C. In diabetic patients with proteinuria and declining renal function,
lowering the systemic blood pressure slows the rate of decline in renal
function.
D. In patients with microalbuminuria, using ACE inhibitors decreases urine
albumin excretion and rate progression to overt nephropathy.
E. Avoiding NSAIDs and radiocontrast media, and properly managing UTIs and
neurogenic bladder, are important measures to preserve the remaining renal

function in patients with established diabetic nephropathy.

Show answer
Correct Answer: A

51. A 22-year- old man presents with the complaint of recurrent blood in his
urine. He states that it usually occurs when he gets "the flu". He has no
other complaints and he has been in good health otherwise. He is not taking
any medication and has no drug allergies. Physical examination including B.P.
is normal and he has no edema. Urinalysis reveals trace proteinuria, several
dysmorphic RBCs and few RBC casts. BUN is 12 mg/dl and serum creatinine 0.6
mg/dl. Which one of the following is the most likely diagnosis?
A. Nephrolithiasis
B. Minimal change disease
C. IgA nephropathy
D. Focal segmental glomerulosclerosis

Show answer
Correct Answer: C

52. A 7-year-old girl presents with leg edema. She has no other complaint and
has been healthy up until now. She is not taking any medication and has no
known allergies. Physical examination shows 3+ leg edema. The rest of the
physical examination including B.P. is normal. Urinalysis reveals 3+
proteinuria , no RBC, WBC or any casts. BUN and serum creatinine are normal.
Serum albumin is 1.5 mg/dl. 24-hour urine collection shows 6.0 gram protein.
Which one of the following statements about her disease is true?
A. Renal biopsy will most likely reveal extensive crescent formation in the
glomeruli.
B. This is the most common cause of nephrotic syndrome in children.
C. She has about 90% chance of developing end stage renal disease.
D. This disease is highly resistant to steroids.

Show answer
Correct Answer: B

53. Which one of the following statements about post-streptococcal

glomerulonephritis is true?
A. It occurs simultaneously with the throat infection.
B. It is associated with low serum complement levels.
C. It commonly progresses to end stage renal disease.
D. Steroids are often needed to treat the disease.

Show answer
Correct Answer: B

54. Which one of the following statements about Goodpasture's syndrome is


true?
A. It predominantly occurs in persons older than 70 years.
B. Kidney biopsy is the most direct way to make the diagnosis.
C. Anti-GBM antibody titres correlate well with the disease severity.
D. Plasmapheresis is not an effective therapy.

Show answer
Correct Answer: B

55. All of the following about systemic lupus erythematosus are true except:
A. It is an immune complex vasculitis process that may cause
glomerulonephritis.
B. Women are more commonly affected than men.
C. The glomerular lesions span a spectrum from normal glomeruli to diffuse
necrotizing crescentic glomerulonephritis.
D. Plasmapheresis is the best form of therapy.

Show answer
Correct Answer: D

56. Membranoproliferative glomerulonephritis (MPGN) is characterized by:


A. Consistently normal serum complement levels.
B. No progression to chronic renal failure.
C. Both mesangial hypercellularity and glomerular basement membrane changes.
D. Steroids are very effective in adult patients.
E. Absence of proteinuria.

Show answer
Correct Answer: C

57. A middle aged woman has active stone disease for many years. Her stones
are radioopaque, small and occur bilaterally. She has seen a Urologist for
this problem who performed a metabolic work-up that showed normal urinary Ca
and oxalate excretion. She had been treated, unsuccessfully, with a thiazide
diuretic, low sodium diet and high fluid intake. A stone analysis revealed
that her stones consisted of Ca-oxalate and a small amount of uric acid. Which
of the following treatments will likely be effective in the prevention of
further stones?
A. a low Ca diet
B. Allopurinol, a xanthine oxidase inhibitor
C. vitamin C supplement to acidify the urine
D. a high protein diet
E. none of the above

Show answer
Correct Answer: B

58. Distal nephron potassium secretion is increased by which one of the


following?
A. Triamterene
B. Increased distal nephron sodium delivery
C. Decreased extracellular pH
D. Spironolactone
E. Vasopressin

Show answer
Correct Answer: B

59. Generalized edema can occur in the setting of high, normal, or low serum
sodium concentration.
A. True
B. False

Show answer
Correct Answer: A

60. Generalized edema can occur in the setting of high, normal, or low
intravascular volume.
A. True
B. False

Show answer
Correct Answer: A

61. Ascites can be a manifestation of right-sided congestive heart failure.


A. True
B. False

Show answer
Correct Answer: A

62. The quality of the effective arterial blood volume (EABV) does always
positively correlate with the size of the extracellular volume (ECV).
A. True
B. False

Show answer
Correct Answer: B

63. In order to slow the progression to end stage renal disease in patients
with diabetic nephropathy or glomerulonephritis, Calcium channel blockers are
superior to angiotensin converting enzyme inhibitors.
A. True
B. False

Show answer

Correct Answer: B

64. The original disease that caused end stage renal failure does not recur in
the transplanted kidney because patients are treated with immunosuppressants.
A. True
B. False

Show answer
Correct Answer: B

65. Which of the following is incorrect relative to the definition, diagnosis


and prevalence of hypertension
A. a BP cuff that is too large gives an erroneous BP reading
B. the upper normal BP value for a seven year old is < 120/80 mmHg.
C. African Americans experience a higher prevalence of hypertension than
Caucasians
D. the prevalence of hypertension decreases with age

Show answer
Correct Answer: D

66. Which of the following is incorrect relative to the causes of high blood
pressure:
A. Renal parenchymal disease is the single most common cause of persistent
hypertension in the pre-adolescent population
B. Factors thought to raise BP include obesity, high salt intake, physical
inactivity and immoderate alcohol consumption
C. Renal artery stenosis, renal parenchymal disease, endocrine disorders,
pregnancy, and drugs, account for the majority of secondary hypertension
D. A cause for hypertension can be identified in close to 95% of hypertensive
adults, if diligently searched for

Show answer
Correct Answer: D

67. The routine work-up of a patient who is referred to you with established

hypertension should include


A. Urine toxicology screen
B. IVP
C. Echocardiogram
D. Urinalysis, serum electrolytes and creatinine

Show answer
Correct Answer: D

68. The following is true regarding the treatment of essential hypertension:


A. The elderly need always higher doses of diuretics and beta blockers
B. Beta blockers should generally be avoided in those with reactive airway
disease (asthma)
C. Beta blockers and angiotensin-converting enzyme (ACE) inhibitors should be
avoided following myocardial infarction
D. The slow reduction of medication (step-down therapy) in those with
essential hypertension who have been under good control for one year is
contraindicated

Show answer
Correct Answer: B

69. Which statement is incorrect regarding the increase in glomerular


filtration rate (GFR) that occurs after birth?
A. increased renal blood flow contributes to increased GFR in the neonate
B. GFR increases immediately after birth, regardless of gestational age
C. Redistribution of blood flow to the outer renal cortex increases GFR
D. Renal vascular resistance increases while systemic vascular resistance
falls after birth

Show answer
Correct Answer: B

70. Tubular reabsorption of ________ decreases with maturation


A. Sodium
B. Bicarbonate
C. Potassium

D. Phosphate

Show answer
Correct Answer: D

71. A three month old infant born at full term weighs 5 kg and has a length of
50 cm. The serum creatinine is 1.0. Which statement is false?
A. This infant has normal renal function
B. Glomerulogenesis is complete
C. Tubular maturation is incomplete
D. This infant's glomerular filtration rate is approximately 22 mL/1.73 m2/min

Show answer
Correct Answer: A

72. Which of the following statements about congenital nephrotic syndrome of


the Finnish type is false.
A. It is caused by a mutation in the nephrin gene on chromosome 19.
B. Maternal alpha feto protein is increased in the second trimester.
C. Steroid therapy is useless.
D. Proteinuria is present only at age twelve months

Show answer
Correct Answer: D

73. Factors contributing to renal vein thrombosis include all of the following
EXCEPT:
A. Hemodilution
B. Decreased antithrombin Ill levels
C. Increased platelet activation
D. High molecular weight fibrinogen

Show answer
Correct Answer: A

74. Typical findings in atheroembolic renal disease include all EXCEPT:


A. Renal failure
B. Eosinophilia
C. Hypocomplementemia
D. Livedo reticularis
E. Nephrotic range proteinuria

Show answer
Correct Answer: E

75. The most likely diagnosis in a patient presenting with signs and symptoms
of small vessel vasculitis, antineutrophil cytoplasmic antibodies (ANCA) in
the blood and evidence of asthma and eosinophilia is which of the following:
A. Henoch-Schonlein purpura
B. Cryoglobulinemic vasculitis
C. Microscopic polyangiitis
D. Necrotizing granulomatosis (Wegener's)
E. Churg-Strauss syndrome

Show answer
Correct Answer: E

76. A 45 year old physician has a long history of recurrent kidney stones.
Because of his busy schedule as Chief of Medicine he has undergone 11
extracorporeal shock wave lithotripsy (ESWL) treatments to "get rid of the
stones". He states that he can handle his stone problem quite well in this
manner and that he has seen no reason to have a metabolic work-up. He noticed
in recent months, however, that he has to urinate quite often, especially at
night, that he has exertional dyspnea and that he is more fatigued than usual.
All of the following statements apply to his situation, EXCEPT
A. ESWL increases stone activity
B. repeated ESWL decreases stone activity
C. repeated ESWL can cause hypertension and 2?heart failure;
D. repeated ESWL can cause renal insufficiency;
E. despite a good response to ESWL, a metabolic evaluation is essential.

Show answer

Correct Answer: B

77. Which therapeutic measures correlate least with the prevention of Calcium
oxalate stones in patients with idiopathic hypercalciuria?
A. low calcium diet;
B. sustained alkalinization of the urine with bicarbonate;
C. a high sodium and animal protein diet, low water intake
D. large quantities of cranberry juice;
E. low sodium and normal protein intake, a high urine volume, thiazide
diuretic;

Show answer
Correct Answer: C

78. Which one of the following factors contribute to the increase in


peripheral vascular resistance in congestive heart failure?
A. Effective arterial blood volume is increased due to renal salt and water
retention
B. Increased renin-angiotensin-aldosterone, ADH, and sympathetic nerve
activity
C. Splanchnic vasodilatation
D. Decreased actual blood volume leads to activation of baroreceptors
E. Suppression of the renin-angiotensin system

Show answer
Correct Answer: B

79. Which one of the following does not drive potassium into cells?
A. Insulin
B. Increased extracellular pH
C. Epinephrine
D. Glucagon
E. Aldosterone

Show answer
Correct Answer: D

80. The clinical manifestations of hypokalemia include all of the following


except:
A. Decreases digitalis toxicity
B. Predisposes to ventricular fibrillation
C. EKG shows widened QRS complex and U waves
D. Weakness
E. Predisposes to cardiac arrest

Show answer
Correct Answer: A

81. Which one of the following statements is false?


A. The main cause of hypokalemia due to vomiting is loss of K+ in the vomitus
B. Diuretics are one of the most common causes of hypokalemia
C. Renal failure is a common cause of hyperkalemia
D. Hyperkalemia in diabetes mellitus is at least partially due to inadequate
aldosterone formation
E. Copious watery diarrhea is frequently associated with hypokalemia

Show answer
Correct Answer: A

82. An elderly woman develops a urinary tract infection, becomes confused, and
does not eat or drink for three days at her nursing home. She is found to have
a blood pressure of 70/50 mmHg (very low) and her serum Na is elevated to 150
meq/L (NL up to 145). Which one of the following intravenous fluids would you
recommend?
A. One-half normal saline
B. Normal saline
C. 5% dextrose and water
D. Hypertonic saline
E. One-quarter normal saline

Show answer
Correct Answer: B

83. Which one of the following statements about hypo- or hypernatremia is


true?
A. A patient with frank symptoms of hypo- or hypernatremia should have his/her
serum sodium concentrations rapidly (one hour) corrected back to normal levels
B. The brain fully compensates for hypernatremia within 1 hour by making
"idiogenic osmoles"
C. Too rapid correction of hypo- or hypernatremia may cause serious
neurological complications
D. Hyponatremia due to SIADH is most often due to underlying kidney or liver
disease.
E. "Pseudohyponatremia" is as dangerous as true hyponatremia.

Show answer
Correct Answer: C

84. Which of the following is NOT commonly found in chronic urinary


obstruction?
A. Nocturia
B. No symptoms
C. Renal failure
D. Red cell casts in urinary sediment
E. Polyuria

Show answer
Correct Answer: D

85. A 23 year old sexually active woman presents with urinary frequency and
burning on micturition of 24 hours duration. Her urinalysis shows: Yellow,
hazy urine. pH 6. No glucose. Protein trace, blood trace. 20-50 WBC/HPF, 10-20
RBC/HPF. 0-1 squamous epithelial cells and no bacteria. You prescribe
trimethoprim/sulfamethoxazole tablets twice a day. The next day she feels much
better but her urine culture report reads: "Staphylococcus saprophyticus
100,000 cfu/ml." Which of the following statements is true?
A. The antibiotic treatment should be continued
B. The urine sample was contaminated and the results from it are unreliable.
C. The hematuria likely signifies the presence of a stone or tumor.
D. The Staphylococcus saprophyticus is likely a skin contaminant.
E. The urinalysis is a laboratory error.

Show answer
Correct Answer: A

86. A 58 year old woman presents with hematuria and left flank pain. Renal
ultrasound shows marked left hydronephrosis. Serum creatinine and CBC are
normal. Which of the following is the least relevant diagnostic aid?
A. Renal biopsy
B. Renal isotope scan
C. Palpation for inguinal lymph nodes
D. CAT scan of abdomen
E. Pelvic examination and IVP

Show answer
Correct Answer: A

87. A 26 year-old diabetic woman is seen in the ER for sore throat. Rapid
strep test is positive for streptococcal pharyngitis and she was started on
ampicillin 500 mg four times a day. Ten days later, she develops hematuria,
edema and hypertension. Laboratory studies show: serum creatinine 3.6 mg/dl,
WBC 8,700 with 66% PMN, 25% lymphs, 3% monos and 5% eosinophils; Urinalysis:
pH 6.2, protein 2+, blood 3+, 65 RBCs/HPF, 20-30 RBC casts/LPF. Hansel's stain
is negative for eosinophils. The most likely diagnosis would be:
A. Diabetic nephropathy
B. IgA nephropathy
C. Acute interstitial nephritis
D. Acute pyelonephritis
E. Acute post-streptococcal glomerulonephritis

Show answer
Correct Answer: E

88. Interstitial nephritis can be associated with all of the following except:
A. Small kidney size by renal ultrasound
B. Nephritic sediment with RBC casts
C. Increased risk of transitional cell carcinoma

D. Tubulointerstitial fibrosis on renal biopsy


E. Urinary tract obstruction due to papillary necrosis on IVP

Show answer
Correct Answer: B

89. Autosomal dominant polycystic kidney disease is associated with all of the
following except:
A. Hypertension is very common
B. Ultrasound is very useful in establishing the diagnosis
C. Renal failure progresses slowly
D. The minority of the patients have cerebral aneurysms
E. Lipid insoluble (water soluble) antibiotics should be used when the cysts
become infected

Show answer
Correct Answer: E

90. A 10 month old boy presents with status epilepticus. His intoxicated
mother was unable to provide a history. The child quits seizing after being
treated with diazepam. Exam revealed an obtunded child with tachypnea but a NL
blood pressure. Screening labs showed Na 140 mEq/L, K 5.5 mEq/L, total CO2 6
mEq/L, Cl 104 mEq/L, BUN 15 mg/dl, creatinine 0.6 mg/dl, glucose 40 mg/dl, Ca
9.5 mg/dl, Mg 1.4 mg/dl, PO4 5 mg/dl, serum osmolality 350 mosm/L. The
arterial blood gas showed a pH of 7.0, PCO2 25 mmHg. The patient has a:
A. Metabolic acidosis with NL anion gap
B. Respiratory acidosis
C. Metabolic alkalosis
D. Respiratory alkalosis
E. Metabolic acidosis with increased anion gap

Show answer
Correct Answer: E

91. The anion gap in the patient in question 30 is:


A. 12
B. 15

C. 20
D. None of the above

Show answer
Correct Answer: D

92. What is his possible diagnosis:


A. Septic shock with lactic acidosis
B. Intoxication with rubbing alcohol (isopropyl)
C. Diabetic ketoacidosis
D. Intoxication with wood alcohol (methanol)

Show answer
Correct Answer: D

93. An infant presents to a pediatrician for recurrent vomiting and failure to


thrive. The child has been hospitalized by another primary care physician for
recurrent dehydration. However, data are not available. Weight and length are
below the fifth percentile. Exam shows a normal blood pressure with signs of
mild dehydration. Reflexes are markedly increased. The child is hypotonic.
Chemistry panel showed Na 140 mEq/L, K 2.2 mEq/L, CO2 50 mEq/L, Cl 78 mEq/L,
BUN 20 mg/dl, creatinine 1.0 mg/dI, Ca 9.5 mg/dl, Mg 1.7 mg/dl, PO4 2.8 mg/dl.
Arterial blood gas showed pH 7.54, pCO2 60 mmHg, urine chloride was 40 mEq/L
and urinary calcium excretion was elevated. What is the nature of the
acid-base disorder?
A. Respiratory acidosis
B. Metabolic alkalosis
C. Respiratory alkalosis
D. Metabolic acidosis

Show answer
Correct Answer: B

94. The most unlikely diagnosis in the patient in question 33 is:


A. Gitelman's syndrome
B. Bartter's syndrome
C. Renal tubular acidosis

D. Contraction alkalosis

Show answer
Correct Answer: C

95. Which one of the following is clearly associated with autosomal recessive
polycystic kidney disease?
A. Very large cysts throughout the kidney parenchyma, including cortex and
medulla
B. Cerebral aneurysms
C. Gastrointestinal diverticulum
D. Congenital hepatic fibrosis

Show answer
Correct Answer: D

96. Which one of the following is true regarding urinary tract infections
(UTI) in the neonatal period?
A. UTI are equally common in boys and girls
B. UTI are more common in boys
C. UTI are always more common in girls
D. UTIs never occur in neonates

Show answer
Correct Answer: B

97. Hemolytic uremic syndrome in children


A. Is usually non-diarrheal
B. Must always be treated with antibiotics
C. Is always benign
D. Is best treated with plasma infusions
E. None of the above

Show answer
Correct Answer: E

98. Which one of the following regarding glomerular filtration rate (GFR) is
true:
A. Creatinine clearance is seldom used as a marker of GFR in clinical medicine
because it consistently underestimates true GFR
B. Two patients with identical serum creatinine concentration always have the
same GFR
C. In advanced renal failure (GFR <15 mI/mm), 24-hour urea clearance is useful
in conjunction with 24-hour creatinine clearance to estimate GFR
D. For a given individual, an increase of serum creatinine from 8.0 mg/dl to
10.0 mg/dl represents a loss of greater number of nephrons than an increase of
serum creatinine from 1.0 mg/dl to 2.0 mg/dl
E. Muscle mass is the predominant determinant of creatinine clearance

Show answer
Correct Answer: C

99. A high blood urea nitrogen to serum creatinine ratio (> 20:1) is seen in
one of the following conditions:
A. administration of 2 L iv normal saline
B. anabolic state
C. advanced liver failure
D. low protein diet
E. severe congestive heart failure

Show answer
Correct Answer: E

100. Which of the following is most likely to be associated with very low
urinary sodium concentration (< 10 mEq/L):
A. high salt intake in a normal subject
B. severe congestive heart failure
C. furosemide (powerful diuretic) administration
D. acute tubular necrosis

Show answer
Correct Answer: B

101. In a patient with oliguria, which ONE of the following favors the
diagnosis of pre-renal azotemia rather than acute tubular necrosis:
A. ratio of urine osmolality to plasma osmolality = 2.0
B. ratio of urine creatinine concentration to plasma creatinine concentration
< 20
C. fractional excretion of sodium (FENa) < 1%
D. urinary sodium concentration of 40 mEq/L

Show answer
Correct Answer: C

102. A patient reports that he has noticed a decrease in urine output for 5
days. His BUN is 100 mg/dl and his serum creatinine is 7 mg/dl. Which of the
following is LEAST useful in establishing the etiology of his renal failure:
A. urine sodium concentration
B. ultrasound of the kidneys
C. abdominal x-ray (KUB)
D. urinalysis
E. history and physical

Show answer
Correct Answer: C

103. Which of the following is an indication for urgent dialysis for a patient
with renal failure:
A. tiredness
B. dyspnea on exertion
C. sinus tachycardia of 110/min
D. serum creatinine of 12 mg/dL
E. serum potassium of 8.0 mEq/L

Show answer
Correct Answer: E

104. In the management of patients with chronic glomerulonephritis and chronic

renal failure with a serum creatinine of 2.0 mg/dl, which of the following is
false:
A. angiotensin converting enzyme inhibitors are absolutely contraindicated
because they are toxic to the kidneys in renal failure
B. dosage of medications often require adjustment
C. nonsteroidal anti-inflammatory agents (arthritis medications) should not be
used liberally because the chronically diseased kidneys are sensitive to the
toxic effects of these agents
D. superimposed urinary tract obstruction may further aggravate renal
insufficiency
E. sodium restriction is not always necessary

Show answer
Correct Answer: A

105. The most common cause of end stage renal disease in the adult (in the
U.S.A.) is
A. hereditary nephritis
B. chronic glomerulonephritis
C. post-infectious glomerulonephritis
D. chronic pyelonephritis
E. none of the above

Show answer
Correct Answer: E

106. Which of the following is most commonly encountered in a patient with


chronic renal failure and bone biopsy showing osteitis fibrosa cystica:
A. decreased serum alkaline phosphatase level
B. markedly decreased serum phosphorus concentration
C. increased serum 1 ,25(OH)2D3 level
D. high parathormone level
E. increased serum calcium concentration

Show answer
Correct Answer: D

107. Which of the following is the most common complication of advanced


untreated renal failure:
A. anemia
B. leukocytosis
C. increased peripheral nerve conduction
D. gastrointestinal tumors
E. hypotriglyceridemia

Show answer
Correct Answer: A

108. In the evaluation of proteinuria, which of the following is false:


A. Proteinuria as detected by the dipstick methods is quite insensitive, and
may miss albuminuria in early diabetic nephropathy
B. The dipstick method may not detect Bence-Jones proteins in the urine
C. A random urine sample for protein:creatinine ratio often suffices in
following proteinuria in a patient under treatment, as long as there is no
substantial change in muscle mass.
D. Protein excretion in the urine is a good reflection of dietary protein
intake

Show answer
Correct Answer: D

109. Based on the following data, the fractional excretion of sodium (FENa)
can be calculated to be: BUN = 40 mg/dl, 24 urine volume = 2 liters, serum
creatinine = 2 mg/dL, urine creatinine = 40 mg/dL, serum sodium = 150 mEq/dL,
urine sodium = 15 mEq/L
A. 0.2%
B. 0.5%
C. 5%
D. 20%

Show answer
Correct Answer: B

110. Which one of the following statements is correct:

A. Diabetic nephropathy and proteinuria often develop after 10 years duration


of diabetes
B. Patients with type I diabetes and diabetic nephropathy do not have higher
risk of death than those without diabetic nephropathy
C. All patients with diabetic nephropathy have type I DM (diabetes mellitus)
D. If a patient survives 40 years of DM without developing nephropathy, he or
she is at extremely high risk of doing so in the future
E. Microalbuminuria is an insignificant finding in diabetic patients

Show answer
Correct Answer: A

111. Which one of the following statements is not correct?


A. Hyperglycemia leads to the accumulation of advanced glycosylation end
products (AGE's) in tissues in patients with DM
B. AGE's accumulation in tissues parallels the severity of renal disease
C. decreasing intraglomerular pressure preserves the structure and function of
the glomerulus in diabetic patients
D. AGE's are responsible for end organ damage seen in diabetes
E. Hyperlipidemia does not add any significant risk for developing vascular
complications in diabetic patients

Show answer
Correct Answer: E

112. Which one of the following is correct:


A. Early diabetic nephropathy means development of microalbuminuria
B. Normal urine albumin excretion varies between 400-500 mg/24 hr
C. Microalbuminuria in diabetic patients does not predict the development of
frank proteinuria and ESRD
D. Microalbuminuria in diabetic nephropathy patients doesn't become manifest
until serum creatinine is higher than 2.0 mg/dL
E. Once frank proteinuria is established, almost all patients reach ESRD
within one year

Show answer
Correct Answer: A

113. One of the following is not correct:


A. All patients with type I diabetes of greater than 5 years duration should
have an annual screen for microalbuminuria to identify this high risk
population.
B. 15-20% of patients with diabetic nephropathy have diabetic retinopathy
C. The absence of retinopathy should make one suspect a cause of proteinuria
other than diabetic nephropathy.
D. Nephrotic range proteinuria after only 5 years duration of type I DM is
unlikely to be secondary to diabetic nephropathy.
E. If the urinary albumin excretion rate is greater than 30 mg/24 hr in a
single urine collection, this microalbuminuria needs to be confirmed with
additional collections since other causes of microalbuminuria (e.g.,
hypertension, CHF) are common in diabetic

Show answer
Correct Answer: B

114. Regarding the management of diabetes mellitus, all of the following are
correct EXCEPT:
A. Dialysis therapy, in patients with diabetic nephropathy and ESRD is
necessary once the GFR is less than 15 ml/min
B. Tight blood sugar control increases the risk of developing diabetic
nephropathy
C. In diabetic patients with proteinuria and declining renal function,
lowering the systemic blood pressure slows the rate of decline in renal
function
D. In patients with microalbuminuria, using ACE inhibitors decreases urine
albumin excretion and rate of progression to overt nephropathy
E. Avoiding NSAIDs and radiocontrast media, and properly managing UTIs and
neurogenic bladder, are important measures to preserve the remaining renal
function in patients with established diabetic nephropathy

Show answer
Correct Answer: B

115. A 22 year old man presents with the complaint of recurrent blood in his
urine. He states that it usually occurs when he gets "the flu". He has no
other complaints and he has been in good health otherwise. He is not taking
any medication and has no drug allergies. Physical examination including B.P.

is normal and he has no edema. Urinalysis reveals trace proteinuria, several


dysmorphic RBCs and few RBC casts. BUN is 12 mg/dl and serum creatinine 0.6
mg/dl. Which one of the following is the most likely diagnosis?
A. Nephrolithiasis
B. Minimal change disease
C. Hemorrhagic cystitis
D. Focal segmental glomerulosclerosis
E. IgA nephropathy

Show answer
Correct Answer: E

116. A 7 year old girl presents with leg edema. She has no other complaint and
has been healthy up until now. She is not taking any medication and has no
known allergies. Physical examination shows 3+ leg edema. The rest of the
physical examination including B.P. is normal. Urinalysis reveals 3+
proteinuria , no RBC, WBC or any casts. BUN and serum creatinine are normal.
Serum albumin is 1.5 mg/dl. 24 hour urine collection shows 6.0 gram protein.
Which one of the following statements about her disease is true?
A. Renal biopsy will most likely reveal extensive crescent formation in the
glomeruli
B. This is the least common cause of nephrotic syndrome in children
C. She has about 90% chance of developing end stage renal disease
D. This disease is highly responsive to steroids

Show answer
Correct Answer: D

117. Which one of the following statements about post-streptococcal


glomerulonephritis is incorrect?
A. It occurs simultaneously with the throat infection
B. It is associated with low serum complement levels
C. It rarely progresses to end stage renal disease
D. Steroids are not needed to treat the disease

Show answer
Correct Answer: A

118. Which one of the following statements about Goodpasture's syndrome is


true?
A. It predominantly occurs in persons older than 70 years
B. Kidney biopsy is too risky to make the diagnosis
C. Anti-GBM antibody titres do not correlate well with the disease severity
D. Plasmapheresis is not an effective therapy

Show answer
Correct Answer: C

119. All of the following about systemic lupus erythematosus are true except:
A. It is an immune complex vasculitis process that may cause
glomerulonephritis
B. Men are more commonly affected than women
C. The glomerular lesions span a spectrum from normal glomeruli to diffuse
necrotizing crescentic glomerulonephritis
D. Plasmapheresis is rarely indicated

Show answer
Correct Answer: B

120. Membranoproliferative glomerulonephritis (MPGN) is characterized by


A. Consistently normal serum complement levels
B. No progression to chronic renal failure
C. Both mesangial hypercellularity and glomerular basement membrane changes
D. Steroids are very effective in adult patients
E. Absence of proteinuria

Show answer
Correct Answer: C

121. A middle aged woman has active stone disease for many years. Her stones
are radioopaque, small and occur bilaterally. She has seen a Urologist for
this problem who performed a metabolic work-up that showed normal urinary Ca
and oxalate excretion. She had been treated, unsuccessfully, with a thiazide
diuretic, low sodium diet and high fluid intake. A stone analysis revealed

that her stones consisted of Ca-oxalate and a small amount of uric acid. Which
of the following treatments will likely be effective in the prevention of
further stones?
A. a low Ca diet
B. Allopurinol, a xanthine oxidase inhibitor
C. vitamin C supplement to acidify the urine
D. a high protein diet
E. none of the above

Show answer
Correct Answer: B

122. Which one of the following statements about diuretic action is false?
A. Amiloride inhibits apical sodium channels in the cortical collecting tubule
B. Spironolactone inhibits K+ secretion
C. Thiazides inhibit Na+/Cl- entry into the distal tubule
D. Furosemide acts primarily in the proximal tubule
E. Acetazolamide inhibits proximal tubule Na+ reabsorption by reducing
carbonic anhydrase activity

Show answer
Correct Answer: D

123. Distal nephron potassium secretion is decreased by which one of the


following?
A. Furosemide
B. Decreased distal nephron sodium delivery
C. High urine flow
D. Alkaline urine
E. Vasopressin

Show answer
Correct Answer: B

124. Generalized edema can occur only in the setting of low serum sodium
concentration.

A. true
B. false

Show answer
Correct Answer: B

125. Generalized edema can occur only in the setting of high intravascular
volume.
A. true
B. false

Show answer
Correct Answer: B

126. Ascites is a manifestation of isolated left-sided congestive heart


failure?
A. true
B. false

Show answer
Correct Answer: B

127. The quality of the effective arterial blood volume (EABV) never
correlates with the size of the extracellular volume (ECV).
A. true
B. false

Show answer
Correct Answer: B

128. In order to slow the progression to end stage renal disease in patients
with diabetic nephropathy or glomerulonephritis Calcium channel blockers are
inferior to angiotensin converting enzyme inhibitors.
A. true

B. false

Show answer
Correct Answer: A

129. The original disease that caused end stage renal failure does not recur
in the transplanted kidney because patients are treated with
immunosuppressants.
A. true
B. false

Show answer
Correct Answer: B

130. A 79 y/o diabetic woman is brought to the emergency room from a nursing
home because of changes in mental status, obtundation and confusion that began
2-3 days earlier. The patient blood tests show the following: serum sodium
148 meq/L (NL ~ 140), serum potassium 5.7 meq/L (NL 3.5-5.5), BUN 152 mg/dL
(NL 10-20), blood sugar 70 mg/dL (NL 74-12. Which one of the following changes
in serum levels is the most likely explanation for this patient changes in
mental status?
A. hypernatremia
B. hyperkalemia
C. azotemia
D. hypoglycemia

Show answer
Correct Answer: C

131. A 68 y/o, obese woman with type 2 diabetes mellitus x 2 years has had a
significant weight loss, pain in several ribs and her lower back. She has
ankle edema and is found to have lytic bone lesions on x-ray, normal blood and
urinary albumin levels, elevated serum calcium and creatinine, and abnormal
results on serum and urine protein electrophoresis. What is the most likely
diagnosis?
A. minimal change disease
B. lupus nephritis

C. multiple myeloma
D. diabetic nephropathy

Show answer
Correct Answer: C

132. A 17 y/o female patient is diagnosed by her family physician as having


recurrent urinary tract infections over the last 3 months. She is repeatedly
treated with antibiotics because she has microscopic hematuria, proteinuria
and nocturia, occasional low grade fever, some weight loss. However, she never
had dysuria and blood tests and urine cultures were not obtained. She now
presents to the emergency room because she feels weak, looks pale and has
ankle edema and hypertension. Her serum creatinine is 4.0 mg/dL (NL 1.2), the
urinalysis shows large numbers of red cells, red cell casts and 3+ protein,
the hematocrit is 25% (NL ~ 45%). Which serious renal disorder (1) should have
been diagnosed and treated before and (2) is the likely cause of this patient
current condition?
A. Pyelonephritis with papillary necrosiss
B. Rapidly progressive, crescentic glomerulonephritis
C. Acute tubular necrosis
D. Acute, post-streptococcal glomerulonephritis

Show answer
Correct Answer: B

133. Which one of the following kidney stones can be dissolved and their
formation prevented by lowering the urinary pH?
A. uric acid stones
B. calcium-oxalate stones
C. "Struvite" (triple phosphate or infection stones)
D. Cysteine stones

Show answer
Correct Answer: C

134. Linear immunofluorescence along the glomerular wall is characteristic of


which of the following diseases?

A. minimal change disease


B. focal and segmental glomerulosclerosis
C. anti-glomerular basement membrane glomerulonephritis
D. acute postinfectious glomerulonephritis
E. membranous nephropathy

Show answer
Correct Answer: C

135. Normal complement levels (C3/C4) are found in


A. acute postinfectious glomerulonephritis
B. IgA nephropathy
C. lupus glomerulonephritis
D. membranoproliferative glomerulonephritis

Show answer
Correct Answer: B

136. A patient with diabetes mellitus develops nephrotic syndrome. Which one
of the following makes membranous nephropathy a less likely diagnosis?
A. RBCs in the urine
B. 24 hour protein excretion of 12 grams
C. fundoscopic evidence of retinopathy
D. family history of diabetes

Show answer
Correct Answer: C

137. In a patient with nephrotic syndrome, the safest treatment to reduce


proteinuria is
A. high dose prednisone
B. cyclophosphamide
C. calcium channel blocker
D. angiotensin II blocker
E. cyclosporine

Show answer
Correct Answer: D

138. A 35 year-old white man is referred to you for evaluation of microscopic


hematuria detected on routine testing. He has no history of hypertension,
headaches, or urinary tract symptoms. His father and paternal aunt have
polycystic kidney disease; they developed end-stage renal disease at ages 58
and 62 respectively but are currently doing well. The patient has recently
married and is concerned about risks to his future offspring. Blood pressure
is normal. The kidneys are not palpable. Urinalysis shows 8-10 RBCs/hpf and no
protein.
Which of the following studies is most appropriate now?
A. Genetic analysis
B. Renal ultrasonography
C. Cerebral arteriography
D. Intravenous urography

Show answer
Correct Answer: B

139. Two weeks ago, a 27 year-old man was admitted to the hospital because of
progressive fever and weight loss. He had a normal kidney function with a
serum creatinine of 1.2 mg/dL but was found to have positive blood cultures
(4/4) for alpha-hemolytic streptococci and therapy with ceftriaxone was begun.
You are asked to evaluate this patient after two weeks of therapy because
serum creatinine has risen to 3.5 mg/dL. His general condition has improved
(Tmax 37.2 C) but a diffuse maculopapular rash has developed on the trunk.
Repeat blood cultures are negative. The following laboratory results were
obtained: Serum creatinine 3.5 mg/dL; Serum electrolytes (mEq/L): Na 137, K
5.6, Cl 95, bicarb 18; Serum complement C3 150 mg/dL (normal 100-233), CH50
150 units/mL (normal 110-190). Urinalysis showed protein 1+, glucose negative,
blood 3+, many RBCs, WBCs, and WBC casts.
Which of the following is the most likely cause of the elevation in serum
creatinine?
A. Ceftriaxone-associated acute tubular necrosis
B. Glomerulonephritis secondary to subacute bacterial endocarditis
C. Acute interstitial nephritis
D. Intratubular crystal deposition
E. Renal emboli

Show answer
Correct Answer: C

140. Which one of the following disorders is associated with renal papillary
necrosis and irregular-shaped small kidneys?
A. Analgesic nephropathy
B. Diabetic nephropathy
C. Medullary sponge kidney
D. Primary hyperparathyroidism
E. Membranoproliferative glomerulonephritis

Show answer
Correct Answer: A

141. All of the following are features of chronic urinary tract obstructions
except:
A. nocturia
B. hypokalemia
C. acidemia
D. hypertension
E. azotemia

Show answer
Correct Answer: B

142. 80 year old diabetic male had nocturia 1-2 times per night for the last 3
years, but for the last couple of weeks he has to get up 3-4 times per night
to urinate. He is now seen in the emergency room with newly diagnosed renal
failure (serum creatinine 4.8 mg/dl), edema, fever, lower abdominal pain,
hyperkalemia, and hypertension. He has 3+ peripheral edema, normal albumin and
liver enzymes. The most appropriate initial diagnostic test step is
A. fractional excretion of sodium
B. 24 hour collection for creatinine clearance
C. renal ultrasound
D. bladder catheter insertion
E. urinalysis and urine culture

Show answer
Correct Answer: D

143. A patient has the following arterial blood gas results: pH 7.2, [H+] 63
and pCO2 47 mm Hg. The calculated [HCO3-] is:
A. 9 mmol/L
B. 18
C. 24
D. 30
E. 36

Show answer
Correct Answer: B

144. A patient has the following arterial blood gas and venous chemistries: pH
7.2, pCO2 20 mm Hg, [HCO3-] 8 mmol/L, [Na+] 140 mmol/L, [Cl-] 108 mmol/L, [K+]
6 mmol/L. The calculated anion gap is:
A. 12 mmol/L
B. 20
C. 24
D. 26
E. 32

Show answer
Correct Answer: C

145. What type of compensation is expected with metabolic acidosis?


A. Renal compensation results in an increase in [HCO3-] and a decrease in pH.
B. Respiratory compensation results in an increase in pCO2 and a decrease in
pH.
C. Renal compensation results in a decrease in [HCO3-] and a decrease in pH.
D. Respiratory compensation results in a decrease in pCO2 and an increase in
pH.
E. Respiratory compensation results in an increase in pCO2 and an increase in
pH.

Show answer
Correct Answer: D

14
yang
Join Date: 04 Aug 2004
Post Count: 58
Posted on: Fri, 08 Oct 2004, 19:08:36
Post subject:
146. What type of compensation is expected with respiratory acidosis?
A. Renal compensation results in an increase in [HCO3-] and an increase in pH.
B. Respiratory compensation results in a decrease in pCO2 and a decrease in
pH.
C. Respiratory compensation results in a decrease in pCO2 and an increase in
pH.
D. Renal compensation results in an increase in [HCO3-] and a decrease in pH.
E. Respiratory compensation results in a decrease in pCO2 and a decrease in
pH.

Show answer
Correct Answer: A

147. A patient had the following laboratory results: pH 7.25, pCO2 62 mm Hg,
[HCO3-] 26 mmol/L. The patient has what acid base disorder?
A. Metabolic alkalosis
B. Metabolic acidosis
C. Respiratory alkalosis
D. Respiratory acidosis
E. None

Show answer
Correct Answer: D

148. A patient has the following laboratory results: pH 7.5, pCO2 45 mm Hg,

[HCO3-] 34 mmol/L. The patient has what disorder:


A. Metabolic alkalosis
B. Metabolic acidosis
C. Respiratory alkalosis
D. Respiratory acidosis
E. None

Show answer
Correct Answer: A

149. A patient has the following test results. Blood testing shows the
following: pH 7.2, pCO2 20 mm Hg, [HCO3-] 8 mmol/L, [Na+] 140 mmol/L, [Cl-]
122 mmol/L, [K+] 3.0 mmol/L. The urine pH was 7.0 and the urine net charge was
positive. The urinalysis showed no glucose or ketones. Which diagnosis is most
plausible:
A. Proximal renal tubular acidosis.
B. Distal renal tubular acidosis
C. Type IV renal tubular acidosis
D. Diabetic ketoacidosis
E. Methanol ingestion

Show answer
Correct Answer: B

150. A patient presents with the following test results. Blood test results
included pH 7.55, pCO2 50 mm Hg, [HCO3-] 42 mmol/L, [Na+] 140 mmol/L, [K+] 2.5
mmol/L [Cl-] 86 mmol/L. Urine test results included pH 8.0 and [Cl-] 5 mmol/L.
A urine drug screen is negative. The most likely explanation is:
A. Bartter syndrome
B. Renal tubular acidosis
C. Diabetic ketoacidosis
D. Diuretic treatment
E. Vomiting

Show answer
Correct Answer: E

151. A hypotensive patient with a normal serum sodium concentration is treated


with intravenous fluids. Which one of the following intravenous fluids would
you recommend?
A. Isotonic saline
B. Hypotonic saline
C. Hypertonic saline
D. 5% dextrose in water
E. water

Show answer
Correct Answer: A

152. Which one of the following statements is true about hyponatremia?


A. Hyponatremia in the setting of a low urinary sodium concentration is always
associated with extracellular fluid volume depletion.
B. SIADH and primary polydipsia associated hyponatremia are due to an increase
in body sodium and water, but the water increase exceeds that of the sodium
increase.
C. Hyponatremia associated with extracellular fluid volume depletion can be
due to both primary renal or extrarenal losses of sodium and water.
D. Hyponatremia is always treated with water restriction.
E. The urine sodium concentration is always high in hyponatremia.

Show answer
Correct Answer: C

153. The criteria for diagnosis of SIADH are:


A. Low serum osmolality and hyponatremia
B. Urine which is less than maximally dilute
C. Urine sodium matches intake
D. Absence of other disorders impairing urine dilution and improvement in
hyponatremia after water restriction
E. All of the above

Show answer
Correct Answer: E

154. A 75 year old woman develops altered mental status and a serum sodium
concentration that is markedly elevated at 165 mEq/L. She had surgery for a
hip fracture several days ago and her serum sodium concentration was normal at
the time of admission for the surgery. She is making about 200 ml of urine a
day. Her GFR is 50% of normal. She is only taking narcotics for pain and SQ
heparin. A Dobhoff tube was placed earlier today to start feeding, but she has
not received anything through it as of yet. The most likely cause of her
hypernatremia is which one of the following?
A. Renal insufficiency induced nephrogenic diabetes insipidus
B. Central diabetes insipidus due to her hip surgery
C. Administration of intravenous normal saline without administration of any
free water
D. Physical inability to reach water coupled with altered thirst due to pain
medications
E. Hyperaldosteronism

Show answer
Correct Answer: D

155. Which one of the following statements is true about edema formation?
A. It can only occur when effective arterial blood volume is reduced.
B. It can only occur in the setting of elevated circulating aldosterone and
vasopressin levels.
C. Increased vascular permeability to proteins is a common factor in most
cases of edema formation.
D. Effective arterial blood volume can not be reduced in the setting of high
actual blood volume.
E. Enhanced proximal and distal nephron sodium and water reabsorption is a
common feature of edema formation in the setting of reduced effective arterial
blood volume.

Show answer
Correct Answer: E

156. A patient is seen with marked edema and a 24 hr urine protein excretion
of 12 grams. The serum albumin concentration is modestly reduced and the blood
pressure is 180/120. GFR is 90% of normal. The edema formation in this patient
is most likely due to which one of the following?
A. Reduced GFR
B. Hepatic dysfunction

C. Peripheral vasodilation
D. An intrinsic renal defect in sodium and water excretion
E. Poor cardiac output

Show answer
Correct Answer: D

157. Which one of the following factors stimulates cellular uptake of


potassium?
A. ?adrenergic antagonists
B. Insulin
C. Acidemia
D. Mineralocorticoid antagonists
E. Vasopressin

Show answer
Correct Answer: B

158. Distal nephron potassium secretion is increased by which one of the


following factors?
A. Hypokalemia
B. Triamterene or amiloride
C. Increased tubule fluid flow rate
D. Spironolactone
E. Acidemia

Show answer
Correct Answer: C

159. A 50 year old male with a 20 year history of diabetes is referred to you
for evaluation of serum creatinine of 4.0 mg/dl. He was seen by an
ophthalmologist last month and was told that his eyes were "clean" on
fundoscopic examination. Two months ago his serum creatinine was 1.0 mg/dl. On
examination his BP was 160/100 mm Hg. Fundus showed no evidence of diabetic
retinopathy. Urine analysis showed 2+ blood and 3+ protein on dipstick. Urine
microscopy showed dysmorphic RBCs. Which of the following statements is true?
A. The most likely diagnosis is diabetic nephropathy

B. This patient requires urgent kidney biopsy


C. The rate of raise in serum creatinine seen in this patient is typical of
diabetic nephropathy
D. here is no correlation between the presence of diabetic retinopathy and
nephropathy
E. Nephrotic range proteinuria is never seen in diabetes.

Show answer
Correct Answer: B

160. Oval fat bodies are seen in


A. Acute tubular necrosis
B. Minimal change disease
C. Papillary necrosis
D. Loin pain hematuria syndrome

Show answer
Correct Answer: B

161. A 70 year old male was admitted to the hospital for osteomyelitis. On
admission, his serum creatinine was 0.8 mg/dl. He was started on Clindamycin
and gentamicin iv and ibuprofen 600 qid for pain. He underwent surgical
debridement on day 2 of hospitalization. Seven days later his serum creatinine
was 3.0 mg/dl. Dipstick was negative for blood and protein. His urine
microscopy showed muddy brown casts and 1-3 RBCs and WBC per hpf. The most
likely diagnosis is
A. Postinfectious glomerulonephritis
B. Acute interstitial nephritis
C. Acute tubular necrosis
D. Dehydration
E. Multiple myeloma

Show answer
Correct Answer: C

162. Vesicoureteral reflux (VUR) in children:


A. Never resolves spontaneously

B. Always requires surgery for correction


C. Is usually a developmental problem with resolution over time
D. None of the above

Show answer
Correct Answer: C

163. Juvenile nephronophthisis is associated with all of the following except:


A. Anemia
B. Hypertension
C. Poor growth
D. Polyuria

Show answer
Correct Answer: B

164. Which of the following does NOT fall into the pre-renal causes of acute
renal failure?
A. Congestive heart failure
B. Sequestration of fluid in "third space" following extensive burn injury
C. Acute renal vasoconstriction from nonsteroidal anti-inflammatory drugs
D. Aminoglycoside nephrotoxicity
E. Acute gastrointestinal hemorrhage

Show answer
Correct Answer: D

165. Which of the following is TRUE regarding acute tubular necrosis (ATN)?
A. Large volume fluid resuscitation is helpful in all phases of ATN in
improving renal function
B. The duration of the oliguric maintenance phase of ATN can last as short as
24 hours and as long as 24 days; recovery in renal function is still possible
under these circumstances
C. Oliguria (less than 500 ml of urine per day) is a prerequisite for ATN
D. Transient polyuria (> 6 liters of urine per day) is a consistent feature
during the recovery from ATN

E. After an episode of ATN, the patient always recovers his/her renal function
completely

Show answer
Correct Answer: B

166. Which of the following is FALSE regarding acute renal failure?


A. History is often helpful in establishing the etiology
B. Acute renal failure is a laboratory diagnosis and often cannot be
ascertained by clinical assessment alone
C. Clinical acute renal failure is often multifactorial
D. If the patient is anuric and in the absence of creatinine loss from other
parts of the body, serum creatinine concentration must increase with time
E. Adjustment of medication dosages is unnecessary in acute renal failure

Show answer
Correct Answer: E

167. Which is the LEAST life-threatening complication of acute renal failure?


A. Hypoxemia from pulmonary edema as a result of fluid overload
B. Blood urea nitrogen concentration over 100 mg/dl
C. Hyperkalemic changes on electrocardiogram
D. Seizure
E. Pericarditis from uremia

Show answer
Correct Answer: B

168. Which of the following does NOT mediate or exacerbate chronic renal
failure?
A. Matrix degradation in the glomeruli
B. Long term uncontrolled hypertension
C. Chronic usage of nonsteroidal anti-inflammatory agents
D. Loss of the majority of the nephron mass
E. Overactivity of angiotensin II locally in the glomeruli

Show answer
Correct Answer: A

169. Which of the following is FALSE regarding chronic renal failure?


A. Renal function may continue to deteriorate despite the apparent
disappearance of immunologic insult from lupus nephritis
B. Acute renal failure can superimpose on chronic renal failure
C. serum creatinine value of less than 1 mg/dl rules out the presence of
impaired glomerular filtration rate (i.e., rules out chronic renal failure)
D. Chronic renal failure is often clinically silent until the impairment in
renal function becomes severe (e.g., glomerular filtration rate below 10% of
normal)
E. Chronic renal failure is often multifactorial

Show answer
Correct Answer: C

170. Which of the following is the LEAST important in the management of


chronic renal failure?
A. Rigorous control of systemic blood pressure
B. Lowering of serum calcium
C. Adjustment of medication dosages because some medications may accumulate in
the body as a result of chronic renal failure
D. Treatment with angiotensin blockers
E. Avoiding volume depletion

Show answer
Correct Answer: B

171. Which of the following is NOT a recognized complication of chronic renal


failure?
A. Decreased production of erythropoietin by the kidneys resulting in
normochromic normocytic anemia
B. Increased aggregation by uremic platelets
C. Autonomic neuropathy leading to dysregulation of blood pressure and
impotence
D. Diffuse inflammation of the gastrointestinal tract
E. Left ventricular hypertrophy of the heart

Show answer
Correct Answer: B

172. Which of the following provides the LEAST accurate estimation of


glomerular filtration rate when GFR is 50 cc/min?
A. Iothalamate clearance
B. Creatinine clearance
C. Urea clearance
D. Average of urea and creatinine clearance

Show answer
Correct Answer: C

173. Which one of the following is FALSE regarding the fractional excretion of
sodium (FENa)?
A. It provides a better indication of the avidity of the renal tubules to
reabsorb sodium than urinary sodium concentration alone
B. Its calculation requires the collection of a 24-hr urine sample
C. Its calculation requires a serum creatinine value
D. Its calculation requires a serum sodium value
E. It is often used clinically to aid in the diagnosis of pre-renal azotemia

Show answer
Correct Answer: B

174. Which one of the following is FALSE regarding urinary electrolytes?


A. It can vary within a wider range than the corresponding serum value in
order to control the serum value tightly
B. It should be interpreted in conjunction with the corresponding serum value
C. It can be useful in the diagnosis of the etiology of serum electrolyte
disorders
D. Random urine samples should not be used because they lack accuracy

Show answer

Correct Answer: D

175. Which of the following imaging procedures should be avoided in a patient


with severe chronic renal failure?
A. Renal ultrasound
B. Magnetic resonance imaging of the kidneys
C. Retrograde pyelogram
D. Radionucleotide renal angiography
E. Intravenous pyelogram

Show answer
Correct Answer: E

176. When measuring blood pressure:


A. Arm and leg blood pressures should be equal.
B. The cuff should be centered over the vein..
C. The bladder width of the cuff should be at least 2/3 of the length of the
extremity being measured.
D. All of the above
E. None of the above

Show answer
Correct Answer: C

177. Blood pressure is a product of:


A. cardiac output and systemic vascular resistance.
B. systemic vascular resistance and weight.
C. height and cardiac output.
D. weight and cardiac output
E. height and weight

Show answer
Correct Answer: A

178. Which statement is true regarding hypertension?


A. Secondary hypertension is more common in the elderly.

B. Elderly patients are particularly prone to diastolic hypertension.


C. Diastolic hypertension is more serious than systolic hypertension.
D. Systolic hypertension is more serious than diastolic hypertension.
E. Step-down therapy, i.e., a slow reduction of medication, can be attempted
in patients with essential hypertension who have been under good control for
one year.

Show answer
Correct Answer: E

179. Which of the following statements are true?


A. Blood pressure classification depends upon age and size.
B. The incidence of hypertension increases with age.
C. Children are more likely to have secondary hypertension than primary
hypertension.
D. All of the above.
E. None of the above.

Show answer
Correct Answer: D

180. Treatment of hypertension often involves:


A. Sedentary lifestyle
B. diuretic therapy
C. beta agonists
D. weight gain
E. angiotensin administration

Show answer
Correct Answer: B

181. T F A healthy 41 y/o lawyer, weighing 70 kg is asked to collect a


complete 24 hour urine in order to determine his creatinine clearance for a
life insurance examination. The test results show, however, that the patient
24 hour urinary creatinine excretion was only 40% of what would be expected in
a patient of this size. Although the lawyer denies such a possibility, the
attending nephrologist states that this result is most likely due to

undercollection and less likely caused by acute renal failure.


A. True
B. False

Show answer
Correct Answer: A

182. T F A 63 y/o male patient with decompensated alcoholic cirrhosis and


tense ascites is admitted to the hospital for "aggressive" diuresis in order
to reduce his abdominal discomfort and shortness of breath. He is given
several high doses of i.v. furosemide (loop diuretic) which elicit a
significant increase in urine output. The patient becomes progressively more
confused, almost comatose and has asterixis. Interpretation: The diuretic
treatment induced hypokalemia and thereby stimulates renal ammoniagenesis
which, in turn, precipitates hepatic encephalopathy.
A. True
B. False

Show answer
Correct Answer: A

183. T F A 21 y/o very depressed student is brought to the ER because of a


suspected suicide attempt. An empty container of digoxin tablets, taken from
his mother who has congestive heart failure and atrial fibrillation, was found
next to his suicide note. The patient serum potassium is 8.6 meq/L (NL 4.5
meq/L). Analysis: Ingestion of a large dose of digoxin causes inhibition of
the Na-K-ATPase and thereby cellular release of potassium resulting in
hyperkalemia. This type of hyperkalemia needs to be immediately treated with
repeated intravenous doses of Calcium-Gluconate because this will reduce
digitalis toxicity.
A. True
B. False

Show answer
Correct Answer: B

184. T F The urine of a 32 y/o man with known cystinuria contains diagnostic

coffin lid-shaped crystals.


A. True
B. False

Show answer
Correct Answer: B

185. T F Extracorporeal Shock Wave Lithotripsy (ESWL) is best suited to treat


large, infected staghorn calculi because it avoids surgical treatment.
A. True
B. False

Show answer
Correct Answer: B

186. T F Gross glomerular hematuria in an adult patient has no impact on


urinary protein excretion because all serum proteins clot in the urinary
system.
A. True
B. False

Show answer
Correct Answer: B

187. T F Expansion of the mesangial matrix is an important pathogenic event in


the development of diabetic nephropathy.
A. True
B. False

Show answer
Correct Answer: A

188. T F Nephrotic syndrome is a risk factor for the development of end stage
renal disease in patients with glomerulonephritis.

A. True
B. False

Show answer
Correct Answer: A

189. T F A positive serum test for anti-neutrophil cytoplasmic antibodies


(ANCA) confirms a diagnosis of vasculitis and rapidly progressive
glomerulonephritis.
A. True
B. False

Show answer
Correct Answer: B

190. A worried 20 year old woman comes to your office. Her mother and aunt
both developed renal failure at age 35-40, and her grandfather had idney
trouble?and died suddenly at age 40 after several years of severe headaches.
She wishes to become pregnant. Her exam and labs are entirely normal. A renal
ultrasound shows numerous bilateral cysts. You should:
A. Advise her that she has a recessive disease; if her husband has no cysts,
there is little chance that their children will have renal cysts
B. Advise her that her children have a 50% chance of having cysts, and that
the course is benign and poses no significant risk to her health
C. Begin testing for renal transplantation
D. Send her for a magnetic resonance angiogram (MRA) of the head
E. Perform a renal biopsy

Show answer
Correct Answer: D

191. Which one of the following patients has a tubulo-interstitial disease?


A. 10 year old boy with sudden onset of edema, hypertension and hematuria 2
weeks after a skin infection
B. 30 year old man with deafness, renal failure, and microscopic hematuria
C. 70 year old man with acute renal failure and eosinophilia after starting
nafcillin

D. 25 year old woman with alopecia, arthralgias, malar rash, and renal failure

Show answer
Correct Answer: C

192. A 40 year old man with arthritis has been using ibuprofen (nonsteroidal
anti-inflammatory) every 6 hours. He developed renal failure associated with
nephrotic range proteinuria. What is the most likely glomerular lesion in this
patient?
A. Membranoproliferative glomerulonephritis
B. Minimal change disease
C. IgA nephropathy
D. Diabetic nephropathy
E. Focal segmental glomerulosclerosis

Show answer
Correct Answer: B

193. A 62 y/o female patient with recurrent urinary infections and large,
bilateral "stag horn" calculi on KUB is likely to demonstrate what kind of
renal calculi
A. Calcium oxalate stones
B. Struvite (triple phosphate stones)
C. Calcium phosphate stones
D. Uric acid stones

Show answer
Correct Answer: B

194. Extracorporeal Shock Wave Lithotripsy (ESWL) is best suited to treat


large, infected staghorn calculi because it avoids surgical treatment.
A. True
B. False

Show answer

Correct Answer: B

195. The metabolic work-up of a 35 y/o male patient with very active kidney
stone disease demonstrates that he has calcium oxalate stones due to
idiopathic hypercalciuria. A marked reduction of dietary calcium intake is the
most effective form of preventive therapy.
A. True
B. False

Show answer
Correct Answer: B

196. A 47-year-old man who has had passed renal calculi on three occasions
over the previous six years presents with nocturia for four months, followed
by increasing lethargy. Laboratory results show sodium 134 mEq per liter,
potassium 5.6 mEq per liter, chloride 112 mEq per liter, bicarbonate 12 mEq
per liter. BUN 48 mg/dl and creatinine 4.0 mg/dl.
The most likely diagnosis is:
A. Obstructive uropathy.
B. Focal segmental glomerulosclerosis.
C. Chronic pyelonephritis.
D. Reflux nephropathy.
E. Polycystic kidney disease.

Show answer
Correct Answer: A

197. A 17-year-old girl had recurrent urinary tract infections in childhood,


but has not had any in the last five years. She now presents with
hypertension, chronic renal failure (serum creatinine 4.8 mg / dl), and
proteinuria of 3 gm/day. The most appropriate investigation at this stage is:
A. IVP
B. Renal angiogram
C. MRI of the kidneys
D. Voiding cystourethrogram
E. Blood serologies including ANCA, anti-GBM antibody, and ANA

Show answer
Correct Answer: D

198. A complicated urinary tract infection is:


A. An infection that causes significant psychological distress in the patient.
B. An infection in a patient with an anatomic abnormality of the urinary
tract.
C. An infection with an organism that is particularly resistant to treatment.
D. An infection in which the patient experiences flank pain and fever.
E. An infection that persists for more than three days without treatment.

Show answer
Correct Answer: B

199. Which patient most likely has a UTI?


A. Urinalysis showing 50 squamous epithelial cells/HPF, 1-2 polymorphs/HPF,
and culture of Escherichia coli greater than 100,000/milliliter.
B. Urinalysis showing 2 squamous epithelial cells/HPF, 20 polymorphs/HPF, and
culture of Escherichia coli, Staphylococcus species, and Gardnerella vaginalis
each greater than 100,000/milliliter.
C. Urinalysis showing 2 squamous epithelial cells/HPF, 20 polymorphs/HPF, and
culture of Escherichia coli greater than 100,000/milliliter.
D. Urinalysis showing 2 squamous epithelial cells/HPF, 2 polymorphs/HPF, and
culture of Gardnerella vaginalis greater than 100,000/milliliter.

Show answer
Correct Answer: C

200. Which statement concerning measurement of urine specific gravity is


correct?
A. Dipstick assays measure all urinary solutes and are precise and reliable in
most clinical circumstances.
B. Refractometry is reliable but requires too large a urine volume to be
practical in severely oliguric patients.
C. Specific gravity is falsely low in patients who have recently received
intravenous radiographic contrast.

D. Specific gravity is elevated by the presence of glucose in the urine.


E. Specific gravity is no help in distinguishing acute tubular necrosis from
prerenal azotemia.

Show answer
Correct Answer: D

201. Dysmorphic urinary erythrocytes signify:


A. Glomerular disease.
B. Intravascular hemolysis.
C. Sickle cell disease.
D. Urinary tract tumor or stone.
E. Diabetic nephropathy.

Show answer
Correct Answer: A

202. Renal tubular cells in the urine signify:


A. Acute tubular necrosis.
B. Acute interstitial nephritis.
C. Chronic glomerulonephritis.
D. Any of the above.

Show answer
Correct Answer: D
203. You discover hexagonal crystals and red blood cells in your patient
urine. The most likely diagnosis is:
A. Urinary infection with struvite calculi.
B. Cystinuria.
C. Hyperoxaluria.
D. Gout with hyperuricosuria.
E. Calcium oxalate nephrolithiasis.

Show answer

Correct Answer: B

204. A patient has the following arterial blood gas results: pH 7.25, [H+] 56
nM and pCO2 47 mm Hg. The calculated [HCO3-] is:
A. 9 mmol/L
B. 18
C. 20
D. 27
E. 36

Show answer
Correct Answer: C

205. A patient has the following arterial blood gas and venous chemistries: pH
7.2, pCO2 20 mm Hg, [HCO3-] 8 mmol/L, [Na+] 142 mmol/L, [Cl-] 108 mmol/L, [K+]
6 mmol/L. The calculated anion gap is:
A. 12 mmol/L
B. 20
C. 24
D. 26
E. 32

Show answer
Correct Answer: D

206. What type of compensation is expected with metabolic acidosis?


A. Renal compensation results in an increase in [HCO3-] and a decrease in pH.
B. Respiratory compensation results in an increase in pCO2 and a decrease in
pH.
C. Renal compensation results in a decrease in [HCO3-] and a decrease in pH.
D. Respiratory compensation results in a decrease in pCO2 and an increase in
pH.
E. Respiratory compensation results in an increase in pCO2 and an increase in
pH.

Show answer
Correct Answer: D

207. What type of compensation is expected with respiratory acidosis?


A. Renal compensation results in an increase in [HCO3-] and an increase in pH.
B. Respiratory compensation results in a decrease in pCO2 and a decrease in
pH.
C. Respiratory compensation results in a decrease in pCO2 and an increase in
pH.
D. Renal compensation results in an increase in [HCO3-] and an decrease in pH.
E. Respiratory compensation results in a decrease in pCO2 and a decrease in
pH.

Show answer
Correct Answer: A

208. A patient had the following laboratory results: pH 7.25, pCO2 62 mm Hg,
[HCO3-] 26 mmol/L. The patient has what acid base disorder.
A. Metabolic alkalosis
B. Metabolic acidosis
C. Respiratory alkalosis
D. Respiratory acidosis
E. None

Show answer
Correct Answer: D

209. A patient has the following laboratory results: pH 7.5, pCO2 45 mm Hg,
[HCO3-] 34 mmol/L. The patient has what disorder:
A. Metabolic alkalosis
B. Metabolic acidosis
C. Respiratory alkalosis
D. Respiratory acidosis
E. None

Show answer
Correct Answer: A

210. A patient has the following test results. Blood pH 7.2, pCO2 20 mm Hg,
[HCO3-] 8 mmol/L, [Na+] 140 mmol/L, [Cl-] 122 mmol/L, [K+] 3.0 mmol/L. The
serum phosphorous was 1.6 mg/dl (low) and the tubular reabsorption of
phosphorous was 60% (low). The urine pH was 6.0. The urinalysis showed trace
glucose but no ketones. Which diagnosis is most plausible:
A. Proximal renal tubular acidosis with renal Fanconi syndrome.
B. Distal renal tubular acidosis
C. Type IV renal tubular acidosis
D. Diabetic ketoacidosis
E. Methanol ingestion

Show answer
Correct Answer: A

211. A patient presents with the following test results. Blood test results
included pH 7.55, pCO2 50 mm Hg, [HCO3-] 42 mmol/L, [Na+] 140 mmol/L, [K+] 2.5
mmol/L [Cl-] 86 mmol/L. Urine test results included pH 8.0 and [Cl-] 2 mmol/L.
A urine drug screen is negative. The most likely explanation is:
A. Bartter syndrome
B. Renal tubular acidosis
C. Diabetic ketoacidosis
D. Diuretic treatment
E. Vomiting

Show answer
Correct Answer: E

212. Which disease rarely, if ever, progresses to end stage renal disease?
A. minimal change disease
B. focal and segmental glomerulosclerosis
C. anti-glomerular basement membrane antibody glomerulonephritis
D. IgA Nephropathy
E. membranous nephropathy

Show answer
Correct Answer: A

213. Low complement levels (C3/C4) are found in which disease?


A. focal and segmental glomerulosclerosis
B. IgA nephropathy
C. lupus glomerulonephritis
D. anti-glomerular basement membrane antibody glomerulonephritis

Show answer
Correct Answer: C

214. A patient with diabetes mellitus develops nephrotic syndrome. Which one
of the following makes diabetic nephropathy the most likely diagnosis?
A. RBCs in the urine
B. 24 hour protein excretion of 12 grams
C. fundoscopic evidence of retinopathy
D. family history of diabetes

Show answer
Correct Answer: C

215. Which drug is the least likely to reduce proteinuria in a patient with
nephrotic syndrome?
A. high dose prednisone
B. ACE inhibitor
C. calcium channel blocker
D. angiotensin receptor blocker
E. cyclosporine

Show answer
Correct Answer: C

216. Thickening of the glomerular basement membrane is a late pathogenic event


in the development of membranous nephropathy.
A. True
B. False

Show answer
Correct Answer: A

217. Patients with nephrotic syndrome are at increased risk for developing
coronary artery disease.
A. True
B. False

Show answer
Correct Answer: A

218. A negative serum test for anti-neutrophil cytoplasmic antibodies (ANCA)


and a normal serum complement level exclude a diagnosis of vasculitis and
rapidly progressive glomerulonephritis.
A. True
B. False

Show answer
Correct Answer: B

219. The most likely diagnosis in an 18 year-old female with recurrent


episodes of macroscopic hematuria and persistent proteinuria is lupus
glomerulonephritis.
A. True
B. False

Show answer
Correct Answer: B

220. Which one of the following statements is correct?


A. An asymptomatic patient with a plasma sodium concentration of 154 mEq/L who
is hypotensive due to dehydration should be treated with 1/2 normal saline
instead of normal saline because of concerns over hypernatremia..
B. A patient with markedly altered mental status thought due to a plasma

sodium concentration of 105 mEq/L should be treated with 1/2 normal saline to
raise the plasma sodium concentration gradually.
C. A patient with markedly altered mental status thought due to a plasma
sodium concentration of 105 mEq/L should have the plasma sodium concentration
corrected to normal levels over the next 24 hours.
D. An asymptomatic patient with a plasma sodium concentration of 110 mEq/L
does not need emergent treatment and can be treated with water restriction.
E. A patient with markedly altered mental status and a plasma sodium
concentration of 165 mEq/L should be treated with IV D5W with the goal being
to normalize the plasma concentration over the next 12 hours.

Show answer
Correct Answer: D

221. The syndrome of inappropriate ADH secretion is usually characterized by


all of the following EXCEPT which one?
A. Low serum osmolality and hyponatremia
B. Sodium retention with low urine sodium concentration
C. Urine which is less than maximally dilute
D. Improvement in hyponatremia with water restriction
E. Presence of pulmonary disease, CNS disease, tumors or drugs

Show answer
Correct Answer: B

222. Which one of the following is FALSE with regards to diabetes insipidus?
A. Urine volume is typically large
B. Drugs are a common cause of nephrogenic diabetes insipidus
C. Central diabetes insipidus is commonly caused by trauma, surgery, or is
idiopathic
D. Plasma ADH concentration can be high in patients with nephrogenic diabetes
insipidus
E. Water restriction as a diagnostic maneuver generally can not distinguish
between central and nephrogenic DI

Show answer
Correct Answer: E

223. Which one of the following statements is correct with regards to the
physiology of renal water handling?
A. ADH release by the pituitary is primarily regulated by plasma osmolality
with relatively little contribution by plasma volume.
B. In order to make adequately concentrated urine it is not necessary to have
normal sodium reabsorption by the thick ascending limb.
C. In order to make adequate dilute urine the collecting duct must remain
impermeable to water.
D. ADH can increase urine osmolality by about 50-fold.
E. The thick ascending limb of Henle loop is permeable to water.

Show answer
Correct Answer: C

224. Edema formation in congestive heart failure is due to all of the


following EXCEPT which one:
A. Decreased cardiac output
B. Increased vasopressin release
C. Increased sympathetic nervous system activity
D. Portal hypertension
E. Increased activity of the renin-angiotensin-aldosterone system

Show answer
Correct Answer: D

225. Which one of the following is true about edema formation?


A. An intrinsic defect in renal sodium and water excretion is primarily
responsible for edema formation in the nephrotic syndrome.
B. Renal sodium retention in cirrhosis is not primarily due to splanchnic
vasodilatation.
C. Decreased hepatic albumin synthesis is the major cause of ascites formation
in cirrhosis.
D. Sodium retention in the nephrotic syndrome is almost always associated with
a marked reduction in GFR (<20 ml/min).
E. Hypoalbuminemia is the major factor in the initial sodium retention in the
nephrotic syndrome.

Show answer

Correct Answer: A

226. Which of the following is true about the treatment of edema?


A. The patient will not respond to diuretics if the fractional excretion of
sodium is less than 1%.
B. Intravenous fluids, even at maintenance rates, are rarely a cause of edema
in hospitalized patients.
C. Restricting dietary salt is the most effective way to reduce edema.
D. Diuretics are appropriate agents to give to edematous patients, however
their action may be limited if GFR is markedly reduced.
E. A fractional excretion of sodium greater than 2% is consistent with
congestive heart failure or cirrhosis.

Show answer
Correct Answer: D

227. Which statement is FALSE about the mechanism of edema formation?


A. Edema forming states are usually associated with reduced effective arterial
blood volume.
B. Edema forming states are always associated with increased actual blood
volume.
C. Increased proximal tubule salt and water reabsorption contributes to edema
formation
D. Enhanced distal nephron salt and water reabsorption contributes to edema
formation.
E. Renal blood flow is often reduced in edema-forming states.

Show answer
Correct Answer: B

228. A 40 year old male has diabetes for the past ten years. His blood
pressure is consistently about 140/90 mm Hg. He has 700 mg of protein on a 24
hour urine collection. You want to start him on antihypertensive therapy.
Which of the following would be the best choice in this situation?
A. Beta blocker
B. Calcium channel blocker
C. Thiazide diuretic
D. ACE inhibitors

Show answer
Correct Answer: D

229. Which of the following is not a risk factor for PRIMARY hypertension?
A. African American race
B. High salt intake
C. Renal artery stenosis
D. Obesity

Show answer
Correct Answer: C

230. You are treating a 55 year old obese man for his hypertension who is
tired most of the day. The most likely secondary cause for his hypertension
is:
A. Sleep apnea
B. Cushing syndrome
C. Pheochromocytoma
D. Renal disease

Show answer
Correct Answer: A

231. A low blood urea nitrogen to serum creatinine ratio (<10:1) is most
likely seen in which of the following conditions:
A. catabolic state resulting from sepsis
B. prerenal azotemia
C. high protein diet
D. rhabdomyolysis (muscle breakdown) from ischemia

Show answer
Correct Answer: D

232. For a given individual, an increase of serum creatinine from 5.0 mg/dl to
10.0 mg/dl represents a loss of fewer nephrons than an increase of serum
creatinine from 1.0 mg/dl to 2.0 mg/dl.
A. true
B. false

Show answer
Correct Answer: A

233. Which of the following is most likely to be associated with high urinary
sodium concentration (>20 mEq/L):
A. low salt diet in a normal subject
B. severe congestive heart failure
C. diuretic treatment for edema associated with chronic renal failure
D. renal vasoconstriction from non-steroidal anti-inflammatory agents

Show answer
Correct Answer: C

234. Which of the following is false regarding the fractional excretion of


sodium (FENa)
A. it requires the urinary sodium concentration for calculation
B. it requires the volume of the urine sample for calculation
C. it requires the urinary creatinine concentration for calculation
D. it is useful in distinguishing prerenal azotemia from acute tubular
necrosis

Show answer
Correct Answer: B

235. Which one of the following is not a recognized mechanism of acute renal
failure:
A. decreased renal blood flow as a result of tubulo-glomerular feedback
B. tubular obstruction from casts
C. back-leak of solutes as a result of tubular epithelial cell damage
D. mesangial cell relaxation

Show answer
Correct Answer: D

236. In a patient with oliguria, which one of the following is most consistent
with acute tubular necrosis rather than pre-renal azotemia:
A. urinary sodium concentration = 3 mEq/L
B. ratio of urine osmolality to plasma osmolality = 1.0
C. ratio of urine creatinine concentration to plasma creatinine concentration
= 80
D. fractional excretion of sodium (FENa) = 0.2%

Show answer
Correct Answer: B

237. In a patient with renal failure of uncertain duration and serum


creatinine of 8 mg/dl, which of the following tests is most useful in
assisting in the diagnosis of collecting system obstruction:
A. fractional excretion of sodium
B. ratio of blood urea nitrogen to serum creatinine concentration
C. renal ultrasound
D. intravenous pyelogram
E. renal angiogram with contrast

Show answer
Correct Answer: C

238. Which of the following is an indication for urgent dialysis in a patient


with renal failure:
A. blood urea nitrogen of 120 mg/dL
B. peripheral edema
C. serum potassium of 9.0 mEq/L
D. serum creatinine of 12 mg/dL
E. severe normochromic normocytic anemia with hemoglobin concentration of 7
g/dl.

Show answer

Correct Answer: C

239. Which of the following is false regarding the progression of chronic


renal failure:
A. angiotensin converting enzyme inhibitors and angiotensin receptor blockers
are often beneficial in retarding the progression of the kidney disease
B. kidney disease often progresses even after the initial insult (e.g.,
nonsteroidal anti-inflammatory agents) has been eliminated
C. blood pressure control does not influence the progression of renal disease
D. adjustment of medications is often necessary in order to avoid systemic
toxicity of the medications

Show answer
Correct Answer: C

240. The most common cause of end stage renal disease in the U.S. is:
A. post-infectious glomerulonephritis
B. polycystic kidney disease
C. toxic nephropathy from antibiotics
D. sports injury of kidneys
E. diabetic nephropathy

Show answer
Correct Answer: E

241. Which of the following is not involved in the pathogenesis of osteitis


fibrosa cystica:
A. elevated serum parathyroid hormone level
B. hyperphosphatemia
C. accumulation of beta-2-microglobulin
D. decrease in serum 1,25-dihydroxy-vitamin D3 levels predisposing to
hyperparathyroidism

Show answer
Correct Answer: C

242. Which one of the following is not a typical systemic complication of


chronic renal failure:
A. anemia
B. insulin resistance in patients not known to have diabetes
C. left ventricular hypertrophy
D. decreased serum triglyceride levels
E. neuropathy

Show answer
Correct Answer: D

243. Which is not true of childhood hemolytic uremic syndrome


A. There is usually a diarrheal prodrome
B. It is described by acute renal injury, microangiopathic hemolytic anemia,
and thrombocytopenia
C. Recovery of renal function rarely occurs
D. Neurologic involvement is common

Show answer
Correct Answer: C

244. The most common cause of bladder outlet obstruction in boys is


A. Eagle-Barrett Syndrome (Prune Belly Syndrome)
B. Autosomal recessive polycystic kidney disease
C. Multicystic dysplasia
D. Posterior urethral valves

Show answer
Correct Answer: D

245. Fractional excretion of sodium (FENa) in newborns


A. Is lower in infants than older children and adults
B. Contributes to the negative sodium balance observed in premature infants
C. Is unrelated to gestational age at birth
D. Has never been measured

Show answer
Correct Answer: B

246. Renal dysplasia


A. Is the result of abnormal mesonephric development
B. Is influenced by abnormal inductive effects of the ureteric bud on the
metanephros
C. Describes only one developmental disorder
D. Does not involve primitive elements in the kidney (e.g. cartilage,
primitive tubules)

Show answer
Correct Answer: B
145. What type of compensation is expected with metabolic acidosis?
A. Renal compensation results in an increase in [HCO3-] and a decrease in pH.
B. Respiratory compensation results in an increase in pCO2 and a decrease in
pH.
C. Renal compensation results in a decrease in [HCO3-] and a decrease in pH.
D. Respiratory compensation results in a decrease in pCO2 and an increase in
pH.
E. Respiratory compensation results in an increase in pCO2 and an increase in
pH.

Show answer
Correct Answer: D

146. What type of compensation is expected with respiratory acidosis?


A. Renal compensation results in an increase in [HCO3-] and an increase in pH.
B. Respiratory compensation results in a decrease in pCO2 and a decrease in
pH.
C. Respiratory compensation results in a decrease in pCO2 and an increase in
pH.
D. Renal compensation results in an increase in [HCO3-] and a decrease in pH.
E. Respiratory compensation results in a decrease in pCO2 and a decrease in
pH.

Show answer
Correct Answer: A

147. A patient had the following laboratory results: pH 7.25, pCO2 62 mm Hg,
[HCO3-] 26 mmol/L. The patient has what acid base disorder?
A. Metabolic alkalosis
B. Metabolic acidosis
C. Respiratory alkalosis
D. Respiratory acidosis
E. None

Show answer
Correct Answer: D

148. A patient has the following laboratory results: pH 7.5, pCO2 45 mm Hg,
[HCO3-] 34 mmol/L. The patient has what disorder:
A. Metabolic alkalosis
B. Metabolic acidosis
C. Respiratory alkalosis
D. Respiratory acidosis
E. None

Show answer
Correct Answer: A

149. A patient has the following test results. Blood testing shows the
following: pH 7.2, pCO2 20 mm Hg, [HCO3-] 8 mmol/L, [Na+] 140 mmol/L, [Cl-]
122 mmol/L, [K+] 3.0 mmol/L. The urine pH was 7.0 and the urine net charge was
positive. The urinalysis showed no glucose or ketones. Which diagnosis is most
plausible:
A. Proximal renal tubular acidosis.
B. Distal renal tubular acidosis
C. Type IV renal tubular acidosis
D. Diabetic ketoacidosis
E. Methanol ingestion

Show answer
Correct Answer: B

150. A patient presents with the following test results. Blood test results
included pH 7.55, pCO2 50 mm Hg, [HCO3-] 42 mmol/L, [Na+] 140 mmol/L, [K+] 2.5
mmol/L [Cl-] 86 mmol/L. Urine test results included pH 8.0 and [Cl-] 5 mmol/L.
A urine drug screen is negative. The most likely explanation is:
A. Bartter syndrome
B. Renal tubular acidosis
C. Diabetic ketoacidosis
D. Diuretic treatment
E. Vomiting

Show answer
Correct Answer: E

151. A hypotensive patient with a normal serum sodium concentration is treated


with intravenous fluids. Which one of the following intravenous fluids would
you recommend?
A. Isotonic saline
B. Hypotonic saline
C. Hypertonic saline
D. 5% dextrose in water
E. water

Show answer
Correct Answer: A

152. Which one of the following statements is true about hyponatremia?


A. Hyponatremia in the setting of a low urinary sodium concentration is always
associated with extracellular fluid volume depletion.
B. SIADH and primary polydipsia associated hyponatremia are due to an increase
in body sodium and water, but the water increase exceeds that of the sodium
increase.
C. Hyponatremia associated with extracellular fluid volume depletion can be
due to both primary renal or extrarenal losses of sodium and water.
D. Hyponatremia is always treated with water restriction.
E. The urine sodium concentration is always high in hyponatremia.

Show answer
Correct Answer: C

153. The criteria for diagnosis of SIADH are:


A. Low serum osmolality and hyponatremia
B. Urine which is less than maximally dilute
C. Urine sodium matches intake
D. Absence of other disorders impairing urine dilution and improvement in
hyponatremia after water restriction
E. All of the above

Show answer
Correct Answer: E

154. A 75 year old woman develops altered mental status and a serum sodium
concentration that is markedly elevated at 165 mEq/L. She had surgery for a
hip fracture several days ago and her serum sodium concentration was normal at
the time of admission for the surgery. She is making about 200 ml of urine a
day. Her GFR is 50% of normal. She is only taking narcotics for pain and SQ
heparin. A Dobhoff tube was placed earlier today to start feeding, but she has
not received anything through it as of yet. The most likely cause of her
hypernatremia is which one of the following?
A. Renal insufficiency induced nephrogenic diabetes insipidus
B. Central diabetes insipidus due to her hip surgery
C. Administration of intravenous normal saline without administration of any
free water
D. Physical inability to reach water coupled with altered thirst due to pain
medications
E. Hyperaldosteronism

Show answer
Correct Answer: D

155. Which one of the following statements is true about edema formation?
A. It can only occur when effective arterial blood volume is reduced.
B. It can only occur in the setting of elevated circulating aldosterone and

vasopressin levels.
C. Increased vascular permeability to proteins is a common factor in most
cases of edema formation.
D. Effective arterial blood volume can not be reduced in the setting of high
actual blood volume.
E. Enhanced proximal and distal nephron sodium and water reabsorption is a
common feature of edema formation in the setting of reduced effective arterial
blood volume.

Show answer
Correct Answer: E

156. A patient is seen with marked edema and a 24 hr urine protein excretion
of 12 grams. The serum albumin concentration is modestly reduced and the blood
pressure is 180/120. GFR is 90% of normal. The edema formation in this patient
is most likely due to which one of the following?
A. Reduced GFR
B. Hepatic dysfunction
C. Peripheral vasodilation
D. An intrinsic renal defect in sodium and water excretion
E. Poor cardiac output

Show answer
Correct Answer: D

157. Which one of the following factors stimulates cellular uptake of


potassium?
A. ?adrenergic antagonists
B. Insulin
C. Acidemia
D. Mineralocorticoid antagonists
E. Vasopressin

Show answer
Correct Answer: B

158. Distal nephron potassium secretion is increased by which one of the

following factors?
A. Hypokalemia
B. Triamterene or amiloride
C. Increased tubule fluid flow rate
D. Spironolactone
E. Acidemia

Show answer
Correct Answer: C

159. A 50 year old male with a 20 year history of diabetes is referred to you
for evaluation of serum creatinine of 4.0 mg/dl. He was seen by an
ophthalmologist last month and was told that his eyes were "clean" on
fundoscopic examination. Two months ago his serum creatinine was 1.0 mg/dl. On
examination his BP was 160/100 mm Hg. Fundus showed no evidence of diabetic
retinopathy. Urine analysis showed 2+ blood and 3+ protein on dipstick. Urine
microscopy showed dysmorphic RBCs. Which of the following statements is true?
A. The most likely diagnosis is diabetic nephropathy
B. This patient requires urgent kidney biopsy
C. The rate of raise in serum creatinine seen in this patient is typical of
diabetic nephropathy
D. here is no correlation between the presence of diabetic retinopathy and
nephropathy
E. Nephrotic range proteinuria is never seen in diabetes.

Show answer
Correct Answer: B

160. Oval fat bodies are seen in


A. Acute tubular necrosis
B. Minimal change disease
C. Papillary necrosis
D. Loin pain hematuria syndrome

Show answer
Correct Answer: B

161. A 70 year old male was admitted to the hospital for osteomyelitis. On
admission, his serum creatinine was 0.8 mg/dl. He was started on Clindamycin
and gentamicin iv and ibuprofen 600 qid for pain. He underwent surgical
debridement on day 2 of hospitalization. Seven days later his serum creatinine
was 3.0 mg/dl. Dipstick was negative for blood and protein. His urine
microscopy showed muddy brown casts and 1-3 RBCs and WBC per hpf. The most
likely diagnosis is
A. Postinfectious glomerulonephritis
B. Acute interstitial nephritis
C. Acute tubular necrosis
D. Dehydration
E. Multiple myeloma

Show answer
Correct Answer: C

162. Vesicoureteral reflux (VUR) in children:


A. Never resolves spontaneously
B. Always requires surgery for correction
C. Is usually a developmental problem with resolution over time
D. None of the above

Show answer
Correct Answer: C

163. Juvenile nephronophthisis is associated with all of the following except:


A. Anemia
B. Hypertension
C. Poor growth
D. Polyuria

Show answer
Correct Answer: B

164. Which of the following does NOT fall into the pre-renal causes of acute
renal failure?

A. Congestive heart failure


B. Sequestration of fluid in "third space" following extensive burn injury
C. Acute renal vasoconstriction from nonsteroidal anti-inflammatory drugs
D. Aminoglycoside nephrotoxicity
E. Acute gastrointestinal hemorrhage

Show answer
Correct Answer: D

165. Which of the following is TRUE regarding acute tubular necrosis (ATN)?
A. Large volume fluid resuscitation is helpful in all phases of ATN in
improving renal function
B. The duration of the oliguric maintenance phase of ATN can last as short as
24 hours and as long as 24 days; recovery in renal function is still possible
under these circumstances
C. Oliguria (less than 500 ml of urine per day) is a prerequisite for ATN
D. Transient polyuria (> 6 liters of urine per day) is a consistent feature
during the recovery from ATN
E. After an episode of ATN, the patient always recovers his/her renal function
completely

Show answer
Correct Answer: B

166. Which of the following is FALSE regarding acute renal failure?


A. History is often helpful in establishing the etiology
B. Acute renal failure is a laboratory diagnosis and often cannot be
ascertained by clinical assessment alone
C. Clinical acute renal failure is often multifactorial
D. If the patient is anuric and in the absence of creatinine loss from other
parts of the body, serum creatinine concentration must increase with time
E. Adjustment of medication dosages is unnecessary in acute renal failure

Show answer
Correct Answer: E

167. Which is the LEAST life-threatening complication of acute renal failure?

A. Hypoxemia from pulmonary edema as a result of fluid overload


B. Blood urea nitrogen concentration over 100 mg/dl
C. Hyperkalemic changes on electrocardiogram
D. Seizure
E. Pericarditis from uremia

Show answer
Correct Answer: B

168. Which of the following does NOT mediate or exacerbate chronic renal
failure?
A. Matrix degradation in the glomeruli
B. Long term uncontrolled hypertension
C. Chronic usage of nonsteroidal anti-inflammatory agents
D. Loss of the majority of the nephron mass
E. Overactivity of angiotensin II locally in the glomeruli

Show answer
Correct Answer: A

169. Which of the following is FALSE regarding chronic renal failure?


A. Renal function may continue to deteriorate despite the apparent
disappearance of immunologic insult from lupus nephritis
B. Acute renal failure can superimpose on chronic renal failure
C. serum creatinine value of less than 1 mg/dl rules out the presence of
impaired glomerular filtration rate (i.e., rules out chronic renal failure)
D. Chronic renal failure is often clinically silent until the impairment in
renal function becomes severe (e.g., glomerular filtration rate below 10% of
normal)
E. Chronic renal failure is often multifactorial

Show answer
Correct Answer: C

170. Which of the following is the LEAST important in the management of


chronic renal failure?
A. Rigorous control of systemic blood pressure

B. Lowering of serum calcium


C. Adjustment of medication dosages because some medications may accumulate in
the body as a result of chronic renal failure
D. Treatment with angiotensin blockers
E. Avoiding volume depletion

Show answer
Correct Answer: B

171. Which of the following is NOT a recognized complication of chronic renal


failure?
A. Decreased production of erythropoietin by the kidneys resulting in
normochromic normocytic anemia
B. Increased aggregation by uremic platelets
C. Autonomic neuropathy leading to dysregulation of blood pressure and
impotence
D. Diffuse inflammation of the gastrointestinal tract
E. Left ventricular hypertrophy of the heart

Show answer
Correct Answer: B

172. Which of the following provides the LEAST accurate estimation of


glomerular filtration rate when GFR is 50 cc/min?
A. Iothalamate clearance
B. Creatinine clearance
C. Urea clearance
D. Average of urea and creatinine clearance

Show answer
Correct Answer: C

173. Which one of the following is FALSE regarding the fractional excretion of
sodium (FENa)?
A. It provides a better indication of the avidity of the renal tubules to
reabsorb sodium than urinary sodium concentration alone
B. Its calculation requires the collection of a 24-hr urine sample

C. Its calculation requires a serum creatinine value


D. Its calculation requires a serum sodium value
E. It is often used clinically to aid in the diagnosis of pre-renal azotemia

Show answer
Correct Answer: B

174. Which one of the following is FALSE regarding urinary electrolytes?


A. It can vary within a wider range than the corresponding serum value in
order to control the serum value tightly
B. It should be interpreted in conjunction with the corresponding serum value
C. It can be useful in the diagnosis of the etiology of serum electrolyte
disorders
D. Random urine samples should not be used because they lack accuracy

Show answer
Correct Answer: D

175. Which of the following imaging procedures should be avoided in a patient


with severe chronic renal failure?
A. Renal ultrasound
B. Magnetic resonance imaging of the kidneys
C. Retrograde pyelogram
D. Radionucleotide renal angiography
E. Intravenous pyelogram

Show answer
Correct Answer: E

176. When measuring blood pressure:


A. Arm and leg blood pressures should be equal.
B. The cuff should be centered over the vein..
C. The bladder width of the cuff should be at least 2/3 of the length of the
extremity being measured.
D. All of the above
E. None of the above

Show answer
Correct Answer: C

177. Blood pressure is a product of:


A. cardiac output and systemic vascular resistance.
B. systemic vascular resistance and weight.
C. height and cardiac output.
D. weight and cardiac output
E. height and weight

Show answer
Correct Answer: A

178. Which statement is true regarding hypertension?


A. Secondary hypertension is more common in the elderly.
B. Elderly patients are particularly prone to diastolic hypertension.
C. Diastolic hypertension is more serious than systolic hypertension.
D. Systolic hypertension is more serious than diastolic hypertension.
E. Step-down therapy, i.e., a slow reduction of medication, can be attempted
in patients with essential hypertension who have been under good control for
one year.

Show answer
Correct Answer: E

179. Which of the following statements are true?


A. Blood pressure classification depends upon age and size.
B. The incidence of hypertension increases with age.
C. Children are more likely to have secondary hypertension than primary
hypertension.
D. All of the above.
E. None of the above.

Show answer
Correct Answer: D

180. Treatment of hypertension often involves:


A. Sedentary lifestyle
B. diuretic therapy
C. beta agonists
D. weight gain
E. angiotensin administration

Show answer
Correct Answer: B

181. T F A healthy 41 y/o lawyer, weighing 70 kg is asked to collect a


complete 24 hour urine in order to determine his creatinine clearance for a
life insurance examination. The test results show, however, that the patient
24 hour urinary creatinine excretion was only 40% of what would be expected in
a patient of this size. Although the lawyer denies such a possibility, the
attending nephrologist states that this result is most likely due to
undercollection and less likely caused by acute renal failure.
A. True
B. False

Show answer
Correct Answer: A

182. T F A 63 y/o male patient with decompensated alcoholic cirrhosis and


tense ascites is admitted to the hospital for "aggressive" diuresis in order
to reduce his abdominal discomfort and shortness of breath. He is given
several high doses of i.v. furosemide (loop diuretic) which elicit a
significant increase in urine output. The patient becomes progressively more
confused, almost comatose and has asterixis. Interpretation: The diuretic
treatment induced hypokalemia and thereby stimulates renal ammoniagenesis
which, in turn, precipitates hepatic encephalopathy.
A. True
B. False

Show answer
Correct Answer: A

183. T F A 21 y/o very depressed student is brought to the ER because of a


suspected suicide attempt. An empty container of digoxin tablets, taken from
his mother who has congestive heart failure and atrial fibrillation, was found
next to his suicide note. The patient serum potassium is 8.6 meq/L (NL 4.5
meq/L). Analysis: Ingestion of a large dose of digoxin causes inhibition of
the Na-K-ATPase and thereby cellular release of potassium resulting in
hyperkalemia. This type of hyperkalemia needs to be immediately treated with
repeated intravenous doses of Calcium-Gluconate because this will reduce
digitalis toxicity.
A. True
B. False

Show answer
Correct Answer: B

184. T F The urine of a 32 y/o man with known cystinuria contains diagnostic
coffin lid-shaped crystals.
A. True
B. False

Show answer
Correct Answer: B

185. T F Extracorporeal Shock Wave Lithotripsy (ESWL) is best suited to treat


large, infected staghorn calculi because it avoids surgical treatment.
A. True
B. False

Show answer
Correct Answer: B

186. T F Gross glomerular hematuria in an adult patient has no impact on


urinary protein excretion because all serum proteins clot in the urinary
system.
A. True

B. False

Show answer
Correct Answer: B

187. T F Expansion of the mesangial matrix is an important pathogenic event in


the development of diabetic nephropathy.
A. True
B. False

Show answer
Correct Answer: A

188. T F Nephrotic syndrome is a risk factor for the development of end stage
renal disease in patients with glomerulonephritis.
A. True
B. False

Show answer
Correct Answer: A

189. T F A positive serum test for anti-neutrophil cytoplasmic antibodies


(ANCA) confirms a diagnosis of vasculitis and rapidly progressive
glomerulonephritis.
A. True
B. False

Show answer
Correct Answer: B

190. A worried 20 year old woman comes to your office. Her mother and aunt
both developed renal failure at age 35-40, and her grandfather had idney
trouble?and died suddenly at age 40 after several years of severe headaches.
She wishes to become pregnant. Her exam and labs are entirely normal. A renal

ultrasound shows numerous bilateral cysts. You should:


A. Advise her that she has a recessive disease; if her husband has no cysts,
there is little chance that their children will have renal cysts
B. Advise her that her children have a 50% chance of having cysts, and that
the course is benign and poses no significant risk to her health
C. Begin testing for renal transplantation
D. Send her for a magnetic resonance angiogram (MRA) of the head
E. Perform a renal biopsy

Show answer
Correct Answer: D

191. Which one of the following patients has a tubulo-interstitial disease?


A. 10 year old boy with sudden onset of edema, hypertension and hematuria 2
weeks after a skin infection
B. 30 year old man with deafness, renal failure, and microscopic hematuria
C. 70 year old man with acute renal failure and eosinophilia after starting
nafcillin
D. 25 year old woman with alopecia, arthralgias, malar rash, and renal failure

Show answer
Correct Answer: C

192. A 40 year old man with arthritis has been using ibuprofen (nonsteroidal
anti-inflammatory) every 6 hours. He developed renal failure associated with
nephrotic range proteinuria. What is the most likely glomerular lesion in this
patient?
A. Membranoproliferative glomerulonephritis
B. Minimal change disease
C. IgA nephropathy
D. Diabetic nephropathy
E. Focal segmental glomerulosclerosis

Show answer
Correct Answer: B

193. A 62 y/o female patient with recurrent urinary infections and large,
bilateral "stag horn" calculi on KUB is likely to demonstrate what kind of
renal calculi
A. Calcium oxalate stones
B. Struvite (triple phosphate stones)
C. Calcium phosphate stones
D. Uric acid stones

Show answer
Correct Answer: B

194. Extracorporeal Shock Wave Lithotripsy (ESWL) is best suited to treat


large, infected staghorn calculi because it avoids surgical treatment.
A. True
B. False

Show answer
Correct Answer: B

195. The metabolic work-up of a 35 y/o male patient with very active kidney
stone disease demonstrates that he has calcium oxalate stones due to
idiopathic hypercalciuria. A marked reduction of dietary calcium intake is the
most effective form of preventive therapy.
A. True
B. False

Show answer
Correct Answer: B

196. A 47-year-old man who has had passed renal calculi on three occasions
over the previous six years presents with nocturia for four months, followed
by increasing lethargy. Laboratory results show sodium 134 mEq per liter,
potassium 5.6 mEq per liter, chloride 112 mEq per liter, bicarbonate 12 mEq
per liter. BUN 48 mg/dl and creatinine 4.0 mg/dl.
The most likely diagnosis is:
A. Obstructive uropathy.

B. Focal segmental glomerulosclerosis.


C. Chronic pyelonephritis.
D. Reflux nephropathy.
E. Polycystic kidney disease.

Show answer
Correct Answer: A

197. A 17-year-old girl had recurrent urinary tract infections in childhood,


but has not had any in the last five years. She now presents with
hypertension, chronic renal failure (serum creatinine 4.8 mg / dl), and
proteinuria of 3 gm/day. The most appropriate investigation at this stage is:
A. IVP
B. Renal angiogram
C. MRI of the kidneys
D. Voiding cystourethrogram
E. Blood serologies including ANCA, anti-GBM antibody, and ANA

Show answer
Correct Answer: D

198. A complicated urinary tract infection is:


A. An infection that causes significant psychological distress in the patient.
B. An infection in a patient with an anatomic abnormality of the urinary
tract.
C. An infection with an organism that is particularly resistant to treatment.
D. An infection in which the patient experiences flank pain and fever.
E. An infection that persists for more than three days without treatment.

Show answer
Correct Answer: B

199. Which patient most likely has a UTI?


A. Urinalysis showing 50 squamous epithelial cells/HPF, 1-2 polymorphs/HPF,
and culture of Escherichia coli greater than 100,000/milliliter.
B. Urinalysis showing 2 squamous epithelial cells/HPF, 20 polymorphs/HPF, and

culture of Escherichia coli, Staphylococcus species, and Gardnerella vaginalis


each greater than 100,000/milliliter.
C. Urinalysis showing 2 squamous epithelial cells/HPF, 20 polymorphs/HPF, and
culture of Escherichia coli greater than 100,000/milliliter.
D. Urinalysis showing 2 squamous epithelial cells/HPF, 2 polymorphs/HPF, and
culture of Gardnerella vaginalis greater than 100,000/milliliter.

Show answer
Correct Answer: C

200. Which statement concerning measurement of urine specific gravity is


correct?
A. Dipstick assays measure all urinary solutes and are precise and reliable in
most clinical circumstances.
B. Refractometry is reliable but requires too large a urine volume to be
practical in severely oliguric patients.
C. Specific gravity is falsely low in patients who have recently received
intravenous radiographic contrast.
D. Specific gravity is elevated by the presence of glucose in the urine.
E. Specific gravity is no help in distinguishing acute tubular necrosis from
prerenal azotemia.

Show answer
Correct Answer: D

201. Dysmorphic urinary erythrocytes signify:


A. Glomerular disease.
B. Intravascular hemolysis.
C. Sickle cell disease.
D. Urinary tract tumor or stone.
E. Diabetic nephropathy.

Show answer
Correct Answer: A

202. Renal tubular cells in the urine signify:


A. Acute tubular necrosis.

B. Acute interstitial nephritis.


C. Chronic glomerulonephritis.
D. Any of the above.

Show answer
Correct Answer: D
203. You discover hexagonal crystals and red blood cells in your patient
urine. The most likely diagnosis is:
A. Urinary infection with struvite calculi.
B. Cystinuria.
C. Hyperoxaluria.
D. Gout with hyperuricosuria.
E. Calcium oxalate nephrolithiasis.

Show answer
Correct Answer: B

204. A patient has the following arterial blood gas results: pH 7.25, [H+] 56
nM and pCO2 47 mm Hg. The calculated [HCO3-] is:
A. 9 mmol/L
B. 18
C. 20
D. 27
E. 36

Show answer
Correct Answer: C

205. A patient has the following arterial blood gas and venous chemistries: pH
7.2, pCO2 20 mm Hg, [HCO3-] 8 mmol/L, [Na+] 142 mmol/L, [Cl-] 108 mmol/L, [K+]
6 mmol/L. The calculated anion gap is:
A. 12 mmol/L
B. 20
C. 24
D. 26
E. 32

Show answer
Correct Answer: D

206. What type of compensation is expected with metabolic acidosis?


A. Renal compensation results in an increase in [HCO3-] and a decrease in pH.
B. Respiratory compensation results in an increase in pCO2 and a decrease in
pH.
C. Renal compensation results in a decrease in [HCO3-] and a decrease in pH.
D. Respiratory compensation results in a decrease in pCO2 and an increase in
pH.
E. Respiratory compensation results in an increase in pCO2 and an increase in
pH.

Show answer
Correct Answer: D

207. What type of compensation is expected with respiratory acidosis?


A. Renal compensation results in an increase in [HCO3-] and an increase in pH.
B. Respiratory compensation results in a decrease in pCO2 and a decrease in
pH.
C. Respiratory compensation results in a decrease in pCO2 and an increase in
pH.
D. Renal compensation results in an increase in [HCO3-] and an decrease in pH.
E. Respiratory compensation results in a decrease in pCO2 and a decrease in
pH.

Show answer
Correct Answer: A

208. A patient had the following laboratory results: pH 7.25, pCO2 62 mm Hg,
[HCO3-] 26 mmol/L. The patient has what acid base disorder.
A. Metabolic alkalosis
B. Metabolic acidosis
C. Respiratory alkalosis

D. Respiratory acidosis
E. None

Show answer
Correct Answer: D

209. A patient has the following laboratory results: pH 7.5, pCO2 45 mm Hg,
[HCO3-] 34 mmol/L. The patient has what disorder:
A. Metabolic alkalosis
B. Metabolic acidosis
C. Respiratory alkalosis
D. Respiratory acidosis
E. None

Show answer
Correct Answer: A

210. A patient has the following test results. Blood pH 7.2, pCO2 20 mm Hg,
[HCO3-] 8 mmol/L, [Na+] 140 mmol/L, [Cl-] 122 mmol/L, [K+] 3.0 mmol/L. The
serum phosphorous was 1.6 mg/dl (low) and the tubular reabsorption of
phosphorous was 60% (low). The urine pH was 6.0. The urinalysis showed trace
glucose but no ketones. Which diagnosis is most plausible:
A. Proximal renal tubular acidosis with renal Fanconi syndrome.
B. Distal renal tubular acidosis
C. Type IV renal tubular acidosis
D. Diabetic ketoacidosis
E. Methanol ingestion

Show answer
Correct Answer: A

211. A patient presents with the following test results. Blood test results
included pH 7.55, pCO2 50 mm Hg, [HCO3-] 42 mmol/L, [Na+] 140 mmol/L, [K+] 2.5
mmol/L [Cl-] 86 mmol/L. Urine test results included pH 8.0 and [Cl-] 2 mmol/L.
A urine drug screen is negative. The most likely explanation is:
A. Bartter syndrome
B. Renal tubular acidosis

C. Diabetic ketoacidosis
D. Diuretic treatment
E. Vomiting

Show answer
Correct Answer: E

212. Which disease rarely, if ever, progresses to end stage renal disease?
A. minimal change disease
B. focal and segmental glomerulosclerosis
C. anti-glomerular basement membrane antibody glomerulonephritis
D. IgA Nephropathy
E. membranous nephropathy

Show answer
Correct Answer: A

213. Low complement levels (C3/C4) are found in which disease?


A. focal and segmental glomerulosclerosis
B. IgA nephropathy
C. lupus glomerulonephritis
D. anti-glomerular basement membrane antibody glomerulonephritis

Show answer
Correct Answer: C

214. A patient with diabetes mellitus develops nephrotic syndrome. Which one
of the following makes diabetic nephropathy the most likely diagnosis?
A. RBCs in the urine
B. 24 hour protein excretion of 12 grams
C. fundoscopic evidence of retinopathy
D. family history of diabetes

Show answer
Correct Answer: C

215. Which drug is the least likely to reduce proteinuria in a patient with
nephrotic syndrome?
A. high dose prednisone
B. ACE inhibitor
C. calcium channel blocker
D. angiotensin receptor blocker
E. cyclosporine

Show answer
Correct Answer: C

216. Thickening of the glomerular basement membrane is a late pathogenic event


in the development of membranous nephropathy.
A. True
B. False

Show answer
Correct Answer: A

217. Patients with nephrotic syndrome are at increased risk for developing
coronary artery disease.
A. True
B. False

Show answer
Correct Answer: A

218. A negative serum test for anti-neutrophil cytoplasmic antibodies (ANCA)


and a normal serum complement level exclude a diagnosis of vasculitis and
rapidly progressive glomerulonephritis.
A. True
B. False

Show answer

Correct Answer: B

219. The most likely diagnosis in an 18 year-old female with recurrent


episodes of macroscopic hematuria and persistent proteinuria is lupus
glomerulonephritis.
A. True
B. False

Show answer
Correct Answer: B

220. Which one of the following statements is correct?


A. An asymptomatic patient with a plasma sodium concentration of 154 mEq/L who
is hypotensive due to dehydration should be treated with 1/2 normal saline
instead of normal saline because of concerns over hypernatremia..
B. A patient with markedly altered mental status thought due to a plasma
sodium concentration of 105 mEq/L should be treated with 1/2 normal saline to
raise the plasma sodium concentration gradually.
C. A patient with markedly altered mental status thought due to a plasma
sodium concentration of 105 mEq/L should have the plasma sodium concentration
corrected to normal levels over the next 24 hours.
D. An asymptomatic patient with a plasma sodium concentration of 110 mEq/L
does not need emergent treatment and can be treated with water restriction.
E. A patient with markedly altered mental status and a plasma sodium
concentration of 165 mEq/L should be treated with IV D5W with the goal being
to normalize the plasma concentration over the next 12 hours.

Show answer
Correct Answer: D

221. The syndrome of inappropriate ADH secretion is usually characterized by


all of the following EXCEPT which one?
A. Low serum osmolality and hyponatremia
B. Sodium retention with low urine sodium concentration
C. Urine which is less than maximally dilute
D. Improvement in hyponatremia with water restriction
E. Presence of pulmonary disease, CNS disease, tumors or drugs

Show answer
Correct Answer: B

222. Which one of the following is FALSE with regards to diabetes insipidus?
A. Urine volume is typically large
B. Drugs are a common cause of nephrogenic diabetes insipidus
C. Central diabetes insipidus is commonly caused by trauma, surgery, or is
idiopathic
D. Plasma ADH concentration can be high in patients with nephrogenic diabetes
insipidus
E. Water restriction as a diagnostic maneuver generally can not distinguish
between central and nephrogenic DI

Show answer
Correct Answer: E

223. Which one of the following statements is correct with regards to the
physiology of renal water handling?
A. ADH release by the pituitary is primarily regulated by plasma osmolality
with relatively little contribution by plasma volume.
B. In order to make adequately concentrated urine it is not necessary to have
normal sodium reabsorption by the thick ascending limb.
C. In order to make adequate dilute urine the collecting duct must remain
impermeable to water.
D. ADH can increase urine osmolality by about 50-fold.
E. The thick ascending limb of Henle loop is permeable to water.

Show answer
Correct Answer: C

224. Edema formation in congestive heart failure is due to all of the


following EXCEPT which one:
A. Decreased cardiac output
B. Increased vasopressin release
C. Increased sympathetic nervous system activity
D. Portal hypertension
E. Increased activity of the renin-angiotensin-aldosterone system

Show answer
Correct Answer: D

225. Which one of the following is true about edema formation?


A. An intrinsic defect in renal sodium and water excretion is primarily
responsible for edema formation in the nephrotic syndrome.
B. Renal sodium retention in cirrhosis is not primarily due to splanchnic
vasodilatation.
C. Decreased hepatic albumin synthesis is the major cause of ascites formation
in cirrhosis.
D. Sodium retention in the nephrotic syndrome is almost always associated with
a marked reduction in GFR (<20 ml/min).
E. Hypoalbuminemia is the major factor in the initial sodium retention in the
nephrotic syndrome.

Show answer
Correct Answer: A

226. Which of the following is true about the treatment of edema?


A. The patient will not respond to diuretics if the fractional excretion of
sodium is less than 1%.
B. Intravenous fluids, even at maintenance rates, are rarely a cause of edema
in hospitalized patients.
C. Restricting dietary salt is the most effective way to reduce edema.
D. Diuretics are appropriate agents to give to edematous patients, however
their action may be limited if GFR is markedly reduced.
E. A fractional excretion of sodium greater than 2% is consistent with
congestive heart failure or cirrhosis.

Show answer
Correct Answer: D

227. Which statement is FALSE about the mechanism of edema formation?


A. Edema forming states are usually associated with reduced effective arterial
blood volume.
B. Edema forming states are always associated with increased actual blood

volume.
C. Increased proximal tubule salt and water reabsorption contributes to edema
formation
D. Enhanced distal nephron salt and water reabsorption contributes to edema
formation.
E. Renal blood flow is often reduced in edema-forming states.

Show answer
Correct Answer: B

228. A 40 year old male has diabetes for the past ten years. His blood
pressure is consistently about 140/90 mm Hg. He has 700 mg of protein on a 24
hour urine collection. You want to start him on antihypertensive therapy.
Which of the following would be the best choice in this situation?
A. Beta blocker
B. Calcium channel blocker
C. Thiazide diuretic
D. ACE inhibitors

Show answer
Correct Answer: D

229. Which of the following is not a risk factor for PRIMARY hypertension?
A. African American race
B. High salt intake
C. Renal artery stenosis
D. Obesity

Show answer
Correct Answer: C

230. You are treating a 55 year old obese man for his hypertension who is
tired most of the day. The most likely secondary cause for his hypertension
is:
A. Sleep apnea
B. Cushing syndrome
C. Pheochromocytoma

D. Renal disease

Show answer
Correct Answer: A

231. A low blood urea nitrogen to serum creatinine ratio (<10:1) is most
likely seen in which of the following conditions:
A. catabolic state resulting from sepsis
B. prerenal azotemia
C. high protein diet
D. rhabdomyolysis (muscle breakdown) from ischemia

Show answer
Correct Answer: D

232. For a given individual, an increase of serum creatinine from 5.0 mg/dl to
10.0 mg/dl represents a loss of fewer nephrons than an increase of serum
creatinine from 1.0 mg/dl to 2.0 mg/dl.
A. true
B. false

Show answer
Correct Answer: A

233. Which of the following is most likely to be associated with high urinary
sodium concentration (>20 mEq/L):
A. low salt diet in a normal subject
B. severe congestive heart failure
C. diuretic treatment for edema associated with chronic renal failure
D. renal vasoconstriction from non-steroidal anti-inflammatory agents

Show answer
Correct Answer: C

234. Which of the following is false regarding the fractional excretion of


sodium (FENa)
A. it requires the urinary sodium concentration for calculation
B. it requires the volume of the urine sample for calculation
C. it requires the urinary creatinine concentration for calculation
D. it is useful in distinguishing prerenal azotemia from acute tubular
necrosis

Show answer
Correct Answer: B

235. Which one of the following is not a recognized mechanism of acute renal
failure:
A. decreased renal blood flow as a result of tubulo-glomerular feedback
B. tubular obstruction from casts
C. back-leak of solutes as a result of tubular epithelial cell damage
D. mesangial cell relaxation

Show answer
Correct Answer: D

236. In a patient with oliguria, which one of the following is most consistent
with acute tubular necrosis rather than pre-renal azotemia:
A. urinary sodium concentration = 3 mEq/L
B. ratio of urine osmolality to plasma osmolality = 1.0
C. ratio of urine creatinine concentration to plasma creatinine concentration
= 80
D. fractional excretion of sodium (FENa) = 0.2%

Show answer
Correct Answer: B

237. In a patient with renal failure of uncertain duration and serum


creatinine of 8 mg/dl, which of the following tests is most useful in
assisting in the diagnosis of collecting system obstruction:
A. fractional excretion of sodium
B. ratio of blood urea nitrogen to serum creatinine concentration

C. renal ultrasound
D. intravenous pyelogram
E. renal angiogram with contrast

Show answer
Correct Answer: C

238. Which of the following is an indication for urgent dialysis in a patient


with renal failure:
A. blood urea nitrogen of 120 mg/dL
B. peripheral edema
C. serum potassium of 9.0 mEq/L
D. serum creatinine of 12 mg/dL
E. severe normochromic normocytic anemia with hemoglobin concentration of 7
g/dl.

Show answer
Correct Answer: C

239. Which of the following is false regarding the progression of chronic


renal failure:
A. angiotensin converting enzyme inhibitors and angiotensin receptor blockers
are often beneficial in retarding the progression of the kidney disease
B. kidney disease often progresses even after the initial insult (e.g.,
nonsteroidal anti-inflammatory agents) has been eliminated
C. blood pressure control does not influence the progression of renal disease
D. adjustment of medications is often necessary in order to avoid systemic
toxicity of the medications

Show answer
Correct Answer: C

240. The most common cause of end stage renal disease in the U.S. is:
A. post-infectious glomerulonephritis
B. polycystic kidney disease
C. toxic nephropathy from antibiotics
D. sports injury of kidneys

E. diabetic nephropathy

Show answer
Correct Answer: E

241. Which of the following is not involved in the pathogenesis of osteitis


fibrosa cystica:
A. elevated serum parathyroid hormone level
B. hyperphosphatemia
C. accumulation of beta-2-microglobulin
D. decrease in serum 1,25-dihydroxy-vitamin D3 levels predisposing to
hyperparathyroidism

Show answer
Correct Answer: C

242. Which one of the following is not a typical systemic complication of


chronic renal failure:
A. anemia
B. insulin resistance in patients not known to have diabetes
C. left ventricular hypertrophy
D. decreased serum triglyceride levels
E. neuropathy

Show answer
Correct Answer: D

243. Which is not true of childhood hemolytic uremic syndrome


A. There is usually a diarrheal prodrome
B. It is described by acute renal injury, microangiopathic hemolytic anemia,
and thrombocytopenia
C. Recovery of renal function rarely occurs
D. Neurologic involvement is common

Show answer

Correct Answer: C

244. The most common cause of bladder outlet obstruction in boys is


A. Eagle-Barrett Syndrome (Prune Belly Syndrome)
B. Autosomal recessive polycystic kidney disease
C. Multicystic dysplasia
D. Posterior urethral valves

Show answer
Correct Answer: D

245. Fractional excretion of sodium (FENa) in newborns


A. Is lower in infants than older children and adults
B. Contributes to the negative sodium balance observed in premature infants
C. Is unrelated to gestational age at birth
D. Has never been measured

Show answer
Correct Answer: B

246. Renal dysplasia


A. Is the result of abnormal mesonephric development
B. Is influenced by abnormal inductive effects of the ureteric bud on the
metanephros
C. Describes only one developmental disorder
D. Does not involve primitive elements in the kidney (e.g. cartilage,
primitive tubules)

Show answer
Correct Answer: B
145. What type of compensation is expected with metabolic acidosis?
A. Renal compensation results in an increase in [HCO3-] and a decrease in pH.
B. Respiratory compensation results in an increase in pCO2 and a decrease in
pH.
C. Renal compensation results in a decrease in [HCO3-] and a decrease in pH.
D. Respiratory compensation results in a decrease in pCO2 and an increase in

pH.
E. Respiratory compensation results in an increase in pCO2 and an increase in
pH.

Show answer
Correct Answer: D

146. What type of compensation is expected with respiratory acidosis?


A. Renal compensation results in an increase in [HCO3-] and an increase in pH.
B. Respiratory compensation results in a decrease in pCO2 and a decrease in
pH.
C. Respiratory compensation results in a decrease in pCO2 and an increase in
pH.
D. Renal compensation results in an increase in [HCO3-] and a decrease in pH.
E. Respiratory compensation results in a decrease in pCO2 and a decrease in
pH.

Show answer
Correct Answer: A

147. A patient had the following laboratory results: pH 7.25, pCO2 62 mm Hg,
[HCO3-] 26 mmol/L. The patient has what acid base disorder?
A. Metabolic alkalosis
B. Metabolic acidosis
C. Respiratory alkalosis
D. Respiratory acidosis
E. None

Show answer
Correct Answer: D

148. A patient has the following laboratory results: pH 7.5, pCO2 45 mm Hg,
[HCO3-] 34 mmol/L. The patient has what disorder:
A. Metabolic alkalosis
B. Metabolic acidosis
C. Respiratory alkalosis

D. Respiratory acidosis
E. None

Show answer
Correct Answer: A

149. A patient has the following test results. Blood testing shows the
following: pH 7.2, pCO2 20 mm Hg, [HCO3-] 8 mmol/L, [Na+] 140 mmol/L, [Cl-]
122 mmol/L, [K+] 3.0 mmol/L. The urine pH was 7.0 and the urine net charge was
positive. The urinalysis showed no glucose or ketones. Which diagnosis is most
plausible:
A. Proximal renal tubular acidosis.
B. Distal renal tubular acidosis
C. Type IV renal tubular acidosis
D. Diabetic ketoacidosis
E. Methanol ingestion

Show answer
Correct Answer: B

150. A patient presents with the following test results. Blood test results
included pH 7.55, pCO2 50 mm Hg, [HCO3-] 42 mmol/L, [Na+] 140 mmol/L, [K+] 2.5
mmol/L [Cl-] 86 mmol/L. Urine test results included pH 8.0 and [Cl-] 5 mmol/L.
A urine drug screen is negative. The most likely explanation is:
A. Bartter syndrome
B. Renal tubular acidosis
C. Diabetic ketoacidosis
D. Diuretic treatment
E. Vomiting

Show answer
Correct Answer: E

151. A hypotensive patient with a normal serum sodium concentration is treated


with intravenous fluids. Which one of the following intravenous fluids would
you recommend?
A. Isotonic saline

B. Hypotonic saline
C. Hypertonic saline
D. 5% dextrose in water
E. water

Show answer
Correct Answer: A

152. Which one of the following statements is true about hyponatremia?


A. Hyponatremia in the setting of a low urinary sodium concentration is always
associated with extracellular fluid volume depletion.
B. SIADH and primary polydipsia associated hyponatremia are due to an increase
in body sodium and water, but the water increase exceeds that of the sodium
increase.
C. Hyponatremia associated with extracellular fluid volume depletion can be
due to both primary renal or extrarenal losses of sodium and water.
D. Hyponatremia is always treated with water restriction.
E. The urine sodium concentration is always high in hyponatremia.

Show answer
Correct Answer: C

153. The criteria for diagnosis of SIADH are:


A. Low serum osmolality and hyponatremia
B. Urine which is less than maximally dilute
C. Urine sodium matches intake
D. Absence of other disorders impairing urine dilution and improvement in
hyponatremia after water restriction
E. All of the above

Show answer
Correct Answer: E

154. A 75 year old woman develops altered mental status and a serum sodium
concentration that is markedly elevated at 165 mEq/L. She had surgery for a
hip fracture several days ago and her serum sodium concentration was normal at
the time of admission for the surgery. She is making about 200 ml of urine a

day. Her GFR is 50% of normal. She is only taking narcotics for pain and SQ
heparin. A Dobhoff tube was placed earlier today to start feeding, but she has
not received anything through it as of yet. The most likely cause of her
hypernatremia is which one of the following?
A. Renal insufficiency induced nephrogenic diabetes insipidus
B. Central diabetes insipidus due to her hip surgery
C. Administration of intravenous normal saline without administration of any
free water
D. Physical inability to reach water coupled with altered thirst due to pain
medications
E. Hyperaldosteronism

Show answer
Correct Answer: D

155. Which one of the following statements is true about edema formation?
A. It can only occur when effective arterial blood volume is reduced.
B. It can only occur in the setting of elevated circulating aldosterone and
vasopressin levels.
C. Increased vascular permeability to proteins is a common factor in most
cases of edema formation.
D. Effective arterial blood volume can not be reduced in the setting of high
actual blood volume.
E. Enhanced proximal and distal nephron sodium and water reabsorption is a
common feature of edema formation in the setting of reduced effective arterial
blood volume.

Show answer
Correct Answer: E

156. A patient is seen with marked edema and a 24 hr urine protein excretion
of 12 grams. The serum albumin concentration is modestly reduced and the blood
pressure is 180/120. GFR is 90% of normal. The edema formation in this patient
is most likely due to which one of the following?
A. Reduced GFR
B. Hepatic dysfunction
C. Peripheral vasodilation
D. An intrinsic renal defect in sodium and water excretion
E. Poor cardiac output

Show answer
Correct Answer: D

157. Which one of the following factors stimulates cellular uptake of


potassium?
A. ?adrenergic antagonists
B. Insulin
C. Acidemia
D. Mineralocorticoid antagonists
E. Vasopressin

Show answer
Correct Answer: B

158. Distal nephron potassium secretion is increased by which one of the


following factors?
A. Hypokalemia
B. Triamterene or amiloride
C. Increased tubule fluid flow rate
D. Spironolactone
E. Acidemia

Show answer
Correct Answer: C

159. A 50 year old male with a 20 year history of diabetes is referred to you
for evaluation of serum creatinine of 4.0 mg/dl. He was seen by an
ophthalmologist last month and was told that his eyes were "clean" on
fundoscopic examination. Two months ago his serum creatinine was 1.0 mg/dl. On
examination his BP was 160/100 mm Hg. Fundus showed no evidence of diabetic
retinopathy. Urine analysis showed 2+ blood and 3+ protein on dipstick. Urine
microscopy showed dysmorphic RBCs. Which of the following statements is true?
A. The most likely diagnosis is diabetic nephropathy
B. This patient requires urgent kidney biopsy
C. The rate of raise in serum creatinine seen in this patient is typical of
diabetic nephropathy
D. here is no correlation between the presence of diabetic retinopathy and

nephropathy
E. Nephrotic range proteinuria is never seen in diabetes.

Show answer
Correct Answer: B

160. Oval fat bodies are seen in


A. Acute tubular necrosis
B. Minimal change disease
C. Papillary necrosis
D. Loin pain hematuria syndrome

Show answer
Correct Answer: B

161. A 70 year old male was admitted to the hospital for osteomyelitis. On
admission, his serum creatinine was 0.8 mg/dl. He was started on Clindamycin
and gentamicin iv and ibuprofen 600 qid for pain. He underwent surgical
debridement on day 2 of hospitalization. Seven days later his serum creatinine
was 3.0 mg/dl. Dipstick was negative for blood and protein. His urine
microscopy showed muddy brown casts and 1-3 RBCs and WBC per hpf. The most
likely diagnosis is
A. Postinfectious glomerulonephritis
B. Acute interstitial nephritis
C. Acute tubular necrosis
D. Dehydration
E. Multiple myeloma

Show answer
Correct Answer: C

162. Vesicoureteral reflux (VUR) in children:


A. Never resolves spontaneously
B. Always requires surgery for correction
C. Is usually a developmental problem with resolution over time
D. None of the above

Show answer
Correct Answer: C

163. Juvenile nephronophthisis is associated with all of the following except:


A. Anemia
B. Hypertension
C. Poor growth
D. Polyuria

Show answer
Correct Answer: B

164. Which of the following does NOT fall into the pre-renal causes of acute
renal failure?
A. Congestive heart failure
B. Sequestration of fluid in "third space" following extensive burn injury
C. Acute renal vasoconstriction from nonsteroidal anti-inflammatory drugs
D. Aminoglycoside nephrotoxicity
E. Acute gastrointestinal hemorrhage

Show answer
Correct Answer: D

165. Which of the following is TRUE regarding acute tubular necrosis (ATN)?
A. Large volume fluid resuscitation is helpful in all phases of ATN in
improving renal function
B. The duration of the oliguric maintenance phase of ATN can last as short as
24 hours and as long as 24 days; recovery in renal function is still possible
under these circumstances
C. Oliguria (less than 500 ml of urine per day) is a prerequisite for ATN
D. Transient polyuria (> 6 liters of urine per day) is a consistent feature
during the recovery from ATN
E. After an episode of ATN, the patient always recovers his/her renal function
completely

Show answer
Correct Answer: B

166. Which of the following is FALSE regarding acute renal failure?


A. History is often helpful in establishing the etiology
B. Acute renal failure is a laboratory diagnosis and often cannot be
ascertained by clinical assessment alone
C. Clinical acute renal failure is often multifactorial
D. If the patient is anuric and in the absence of creatinine loss from other
parts of the body, serum creatinine concentration must increase with time
E. Adjustment of medication dosages is unnecessary in acute renal failure

Show answer
Correct Answer: E

167. Which is the LEAST life-threatening complication of acute renal failure?


A. Hypoxemia from pulmonary edema as a result of fluid overload
B. Blood urea nitrogen concentration over 100 mg/dl
C. Hyperkalemic changes on electrocardiogram
D. Seizure
E. Pericarditis from uremia

Show answer
Correct Answer: B

168. Which of the following does NOT mediate or exacerbate chronic renal
failure?
A. Matrix degradation in the glomeruli
B. Long term uncontrolled hypertension
C. Chronic usage of nonsteroidal anti-inflammatory agents
D. Loss of the majority of the nephron mass
E. Overactivity of angiotensin II locally in the glomeruli

Show answer
Correct Answer: A

169. Which of the following is FALSE regarding chronic renal failure?


A. Renal function may continue to deteriorate despite the apparent
disappearance of immunologic insult from lupus nephritis
B. Acute renal failure can superimpose on chronic renal failure
C. serum creatinine value of less than 1 mg/dl rules out the presence of
impaired glomerular filtration rate (i.e., rules out chronic renal failure)
D. Chronic renal failure is often clinically silent until the impairment in
renal function becomes severe (e.g., glomerular filtration rate below 10% of
normal)
E. Chronic renal failure is often multifactorial

Show answer
Correct Answer: C

170. Which of the following is the LEAST important in the management of


chronic renal failure?
A. Rigorous control of systemic blood pressure
B. Lowering of serum calcium
C. Adjustment of medication dosages because some medications may accumulate in
the body as a result of chronic renal failure
D. Treatment with angiotensin blockers
E. Avoiding volume depletion

Show answer
Correct Answer: B

171. Which of the following is NOT a recognized complication of chronic renal


failure?
A. Decreased production of erythropoietin by the kidneys resulting in
normochromic normocytic anemia
B. Increased aggregation by uremic platelets
C. Autonomic neuropathy leading to dysregulation of blood pressure and
impotence
D. Diffuse inflammation of the gastrointestinal tract
E. Left ventricular hypertrophy of the heart

Show answer

Correct Answer: B

172. Which of the following provides the LEAST accurate estimation of


glomerular filtration rate when GFR is 50 cc/min?
A. Iothalamate clearance
B. Creatinine clearance
C. Urea clearance
D. Average of urea and creatinine clearance

Show answer
Correct Answer: C

173. Which one of the following is FALSE regarding the fractional excretion of
sodium (FENa)?
A. It provides a better indication of the avidity of the renal tubules to
reabsorb sodium than urinary sodium concentration alone
B. Its calculation requires the collection of a 24-hr urine sample
C. Its calculation requires a serum creatinine value
D. Its calculation requires a serum sodium value
E. It is often used clinically to aid in the diagnosis of pre-renal azotemia

Show answer
Correct Answer: B

174. Which one of the following is FALSE regarding urinary electrolytes?


A. It can vary within a wider range than the corresponding serum value in
order to control the serum value tightly
B. It should be interpreted in conjunction with the corresponding serum value
C. It can be useful in the diagnosis of the etiology of serum electrolyte
disorders
D. Random urine samples should not be used because they lack accuracy

Show answer
Correct Answer: D

175. Which of the following imaging procedures should be avoided in a patient

with severe chronic renal failure?


A. Renal ultrasound
B. Magnetic resonance imaging of the kidneys
C. Retrograde pyelogram
D. Radionucleotide renal angiography
E. Intravenous pyelogram

Show answer
Correct Answer: E

176. When measuring blood pressure:


A. Arm and leg blood pressures should be equal.
B. The cuff should be centered over the vein..
C. The bladder width of the cuff should be at least 2/3 of the length of the
extremity being measured.
D. All of the above
E. None of the above

Show answer
Correct Answer: C

177. Blood pressure is a product of:


A. cardiac output and systemic vascular resistance.
B. systemic vascular resistance and weight.
C. height and cardiac output.
D. weight and cardiac output
E. height and weight

Show answer
Correct Answer: A

178. Which statement is true regarding hypertension?


A. Secondary hypertension is more common in the elderly.
B. Elderly patients are particularly prone to diastolic hypertension.
C. Diastolic hypertension is more serious than systolic hypertension.
D. Systolic hypertension is more serious than diastolic hypertension.
E. Step-down therapy, i.e., a slow reduction of medication, can be attempted

in patients with essential hypertension who have been under good control for
one year.

Show answer
Correct Answer: E

179. Which of the following statements are true?


A. Blood pressure classification depends upon age and size.
B. The incidence of hypertension increases with age.
C. Children are more likely to have secondary hypertension than primary
hypertension.
D. All of the above.
E. None of the above.

Show answer
Correct Answer: D

180. Treatment of hypertension often involves:


A. Sedentary lifestyle
B. diuretic therapy
C. beta agonists
D. weight gain
E. angiotensin administration

Show answer
Correct Answer: B

181. T F A healthy 41 y/o lawyer, weighing 70 kg is asked to collect a


complete 24 hour urine in order to determine his creatinine clearance for a
life insurance examination. The test results show, however, that the patient
24 hour urinary creatinine excretion was only 40% of what would be expected in
a patient of this size. Although the lawyer denies such a possibility, the
attending nephrologist states that this result is most likely due to
undercollection and less likely caused by acute renal failure.
A. True
B. False

Show answer
Correct Answer: A

182. T F A 63 y/o male patient with decompensated alcoholic cirrhosis and


tense ascites is admitted to the hospital for "aggressive" diuresis in order
to reduce his abdominal discomfort and shortness of breath. He is given
several high doses of i.v. furosemide (loop diuretic) which elicit a
significant increase in urine output. The patient becomes progressively more
confused, almost comatose and has asterixis. Interpretation: The diuretic
treatment induced hypokalemia and thereby stimulates renal ammoniagenesis
which, in turn, precipitates hepatic encephalopathy.
A. True
B. False

Show answer
Correct Answer: A

183. T F A 21 y/o very depressed student is brought to the ER because of a


suspected suicide attempt. An empty container of digoxin tablets, taken from
his mother who has congestive heart failure and atrial fibrillation, was found
next to his suicide note. The patient serum potassium is 8.6 meq/L (NL 4.5
meq/L). Analysis: Ingestion of a large dose of digoxin causes inhibition of
the Na-K-ATPase and thereby cellular release of potassium resulting in
hyperkalemia. This type of hyperkalemia needs to be immediately treated with
repeated intravenous doses of Calcium-Gluconate because this will reduce
digitalis toxicity.
A. True
B. False

Show answer
Correct Answer: B

184. T F The urine of a 32 y/o man with known cystinuria contains diagnostic
coffin lid-shaped crystals.
A. True
B. False

Show answer
Correct Answer: B

185. T F Extracorporeal Shock Wave Lithotripsy (ESWL) is best suited to treat


large, infected staghorn calculi because it avoids surgical treatment.
A. True
B. False

Show answer
Correct Answer: B

186. T F Gross glomerular hematuria in an adult patient has no impact on


urinary protein excretion because all serum proteins clot in the urinary
system.
A. True
B. False

Show answer
Correct Answer: B

187. T F Expansion of the mesangial matrix is an important pathogenic event in


the development of diabetic nephropathy.
A. True
B. False

Show answer
Correct Answer: A

188. T F Nephrotic syndrome is a risk factor for the development of end stage
renal disease in patients with glomerulonephritis.
A. True
B. False

Show answer
Correct Answer: A

189. T F A positive serum test for anti-neutrophil cytoplasmic antibodies


(ANCA) confirms a diagnosis of vasculitis and rapidly progressive
glomerulonephritis.
A. True
B. False

Show answer
Correct Answer: B

190. A worried 20 year old woman comes to your office. Her mother and aunt
both developed renal failure at age 35-40, and her grandfather had idney
trouble?and died suddenly at age 40 after several years of severe headaches.
She wishes to become pregnant. Her exam and labs are entirely normal. A renal
ultrasound shows numerous bilateral cysts. You should:
A. Advise her that she has a recessive disease; if her husband has no cysts,
there is little chance that their children will have renal cysts
B. Advise her that her children have a 50% chance of having cysts, and that
the course is benign and poses no significant risk to her health
C. Begin testing for renal transplantation
D. Send her for a magnetic resonance angiogram (MRA) of the head
E. Perform a renal biopsy

Show answer
Correct Answer: D

191. Which one of the following patients has a tubulo-interstitial disease?


A. 10 year old boy with sudden onset of edema, hypertension and hematuria 2
weeks after a skin infection
B. 30 year old man with deafness, renal failure, and microscopic hematuria
C. 70 year old man with acute renal failure and eosinophilia after starting
nafcillin
D. 25 year old woman with alopecia, arthralgias, malar rash, and renal failure

Show answer
Correct Answer: C

192. A 40 year old man with arthritis has been using ibuprofen (nonsteroidal
anti-inflammatory) every 6 hours. He developed renal failure associated with
nephrotic range proteinuria. What is the most likely glomerular lesion in this
patient?
A. Membranoproliferative glomerulonephritis
B. Minimal change disease
C. IgA nephropathy
D. Diabetic nephropathy
E. Focal segmental glomerulosclerosis

Show answer
Correct Answer: B

193. A 62 y/o female patient with recurrent urinary infections and large,
bilateral "stag horn" calculi on KUB is likely to demonstrate what kind of
renal calculi
A. Calcium oxalate stones
B. Struvite (triple phosphate stones)
C. Calcium phosphate stones
D. Uric acid stones

Show answer
Correct Answer: B

194. Extracorporeal Shock Wave Lithotripsy (ESWL) is best suited to treat


large, infected staghorn calculi because it avoids surgical treatment.
A. True
B. False

Show answer
Correct Answer: B

195. The metabolic work-up of a 35 y/o male patient with very active kidney
stone disease demonstrates that he has calcium oxalate stones due to
idiopathic hypercalciuria. A marked reduction of dietary calcium intake is the
most effective form of preventive therapy.
A. True
B. False

Show answer
Correct Answer: B

196. A 47-year-old man who has had passed renal calculi on three occasions
over the previous six years presents with nocturia for four months, followed
by increasing lethargy. Laboratory results show sodium 134 mEq per liter,
potassium 5.6 mEq per liter, chloride 112 mEq per liter, bicarbonate 12 mEq
per liter. BUN 48 mg/dl and creatinine 4.0 mg/dl.
The most likely diagnosis is:
A. Obstructive uropathy.
B. Focal segmental glomerulosclerosis.
C. Chronic pyelonephritis.
D. Reflux nephropathy.
E. Polycystic kidney disease.

Show answer
Correct Answer: A

197. A 17-year-old girl had recurrent urinary tract infections in childhood,


but has not had any in the last five years. She now presents with
hypertension, chronic renal failure (serum creatinine 4.8 mg / dl), and
proteinuria of 3 gm/day. The most appropriate investigation at this stage is:
A. IVP
B. Renal angiogram
C. MRI of the kidneys
D. Voiding cystourethrogram
E. Blood serologies including ANCA, anti-GBM antibody, and ANA

Show answer
Correct Answer: D

198. A complicated urinary tract infection is:


A. An infection that causes significant psychological distress in the patient.
B. An infection in a patient with an anatomic abnormality of the urinary
tract.
C. An infection with an organism that is particularly resistant to treatment.
D. An infection in which the patient experiences flank pain and fever.
E. An infection that persists for more than three days without treatment.

Show answer
Correct Answer: B

199. Which patient most likely has a UTI?


A. Urinalysis showing 50 squamous epithelial cells/HPF, 1-2 polymorphs/HPF,
and culture of Escherichia coli greater than 100,000/milliliter.
B. Urinalysis showing 2 squamous epithelial cells/HPF, 20 polymorphs/HPF, and
culture of Escherichia coli, Staphylococcus species, and Gardnerella vaginalis
each greater than 100,000/milliliter.
C. Urinalysis showing 2 squamous epithelial cells/HPF, 20 polymorphs/HPF, and
culture of Escherichia coli greater than 100,000/milliliter.
D. Urinalysis showing 2 squamous epithelial cells/HPF, 2 polymorphs/HPF, and
culture of Gardnerella vaginalis greater than 100,000/milliliter.

Show answer
Correct Answer: C

200. Which statement concerning measurement of urine specific gravity is


correct?
A. Dipstick assays measure all urinary solutes and are precise and reliable in
most clinical circumstances.
B. Refractometry is reliable but requires too large a urine volume to be
practical in severely oliguric patients.
C. Specific gravity is falsely low in patients who have recently received
intravenous radiographic contrast.
D. Specific gravity is elevated by the presence of glucose in the urine.
E. Specific gravity is no help in distinguishing acute tubular necrosis from
prerenal azotemia.

Show answer
Correct Answer: D

201. Dysmorphic urinary erythrocytes signify:


A. Glomerular disease.
B. Intravascular hemolysis.
C. Sickle cell disease.
D. Urinary tract tumor or stone.
E. Diabetic nephropathy.

Show answer
Correct Answer: A

Post edited by yang at 08 Oct 2004, 19:11:37.


yang
Join Date: 04 Aug 2004
Post Count: 58
Posted on: Fri, 08 Oct 2004, 19:11:52
Post subject:
203. You discover hexagonal crystals and red blood cells in your patient
urine. The most likely diagnosis is:
A. Urinary infection with struvite calculi.
B. Cystinuria.
C. Hyperoxaluria.
D. Gout with hyperuricosuria.
E. Calcium oxalate nephrolithiasis.

Show answer
Correct Answer: B

204. A patient has the following arterial blood gas results: pH 7.25, [H+] 56
nM and pCO2 47 mm Hg. The calculated [HCO3-] is:
A. 9 mmol/L
B. 18
C. 20

D. 27
E. 36

Show answer
Correct Answer: C

205. A patient has the following arterial blood gas and venous chemistries: pH
7.2, pCO2 20 mm Hg, [HCO3-] 8 mmol/L, [Na+] 142 mmol/L, [Cl-] 108 mmol/L, [K+]
6 mmol/L. The calculated anion gap is:
A. 12 mmol/L
B. 20
C. 24
D. 26
E. 32

Show answer
Correct Answer: D

206. What type of compensation is expected with metabolic acidosis?


A. Renal compensation results in an increase in [HCO3-] and a decrease in pH.
B. Respiratory compensation results in an increase in pCO2 and a decrease in
pH.
C. Renal compensation results in a decrease in [HCO3-] and a decrease in pH.
D. Respiratory compensation results in a decrease in pCO2 and an increase in
pH.
E. Respiratory compensation results in an increase in pCO2 and an increase in
pH.

Show answer
Correct Answer: D

207. What type of compensation is expected with respiratory acidosis?


A. Renal compensation results in an increase in [HCO3-] and an increase in pH.
B. Respiratory compensation results in a decrease in pCO2 and a decrease in
pH.
C. Respiratory compensation results in a decrease in pCO2 and an increase in

pH.
D. Renal compensation results in an increase in [HCO3-] and an decrease in pH.
E. Respiratory compensation results in a decrease in pCO2 and a decrease in
pH.

Show answer
Correct Answer: A

208. A patient had the following laboratory results: pH 7.25, pCO2 62 mm Hg,
[HCO3-] 26 mmol/L. The patient has what acid base disorder.
A. Metabolic alkalosis
B. Metabolic acidosis
C. Respiratory alkalosis
D. Respiratory acidosis
E. None

Show answer
Correct Answer: D

209. A patient has the following laboratory results: pH 7.5, pCO2 45 mm Hg,
[HCO3-] 34 mmol/L. The patient has what disorder:
A. Metabolic alkalosis
B. Metabolic acidosis
C. Respiratory alkalosis
D. Respiratory acidosis
E. None

Show answer
Correct Answer: A

210. A patient has the following test results. Blood pH 7.2, pCO2 20 mm Hg,
[HCO3-] 8 mmol/L, [Na+] 140 mmol/L, [Cl-] 122 mmol/L, [K+] 3.0 mmol/L. The
serum phosphorous was 1.6 mg/dl (low) and the tubular reabsorption of
phosphorous was 60% (low). The urine pH was 6.0. The urinalysis showed trace
glucose but no ketones. Which diagnosis is most plausible:
A. Proximal renal tubular acidosis with renal Fanconi syndrome.

B. Distal renal tubular acidosis


C. Type IV renal tubular acidosis
D. Diabetic ketoacidosis
E. Methanol ingestion

Show answer
Correct Answer: A
211. A patient presents with the following test results. Blood test results
included pH 7.55, pCO2 50 mm Hg, [HCO3-] 42 mmol/L, [Na+] 140 mmol/L, [K+] 2.5
mmol/L [Cl-] 86 mmol/L. Urine test results included pH 8.0 and [Cl-] 2 mmol/L.
A urine drug screen is negative. The most likely explanation is:
A. Bartter syndrome
B. Renal tubular acidosis
C. Diabetic ketoacidosis
D. Diuretic treatment
E. Vomiting

Show answer
Correct Answer: E

212. Which disease rarely, if ever, progresses to end stage renal disease?
A. minimal change disease
B. focal and segmental glomerulosclerosis
C. anti-glomerular basement membrane antibody glomerulonephritis
D. IgA Nephropathy
E. membranous nephropathy

Show answer
Correct Answer: A

213. Low complement levels (C3/C4) are found in which disease?


A. focal and segmental glomerulosclerosis
B. IgA nephropathy
C. lupus glomerulonephritis
D. anti-glomerular basement membrane antibody glomerulonephritis

Show answer

Correct Answer: C

214. A patient with diabetes mellitus develops nephrotic syndrome. Which one
of the following makes diabetic nephropathy the most likely diagnosis?
A. RBCs in the urine
B. 24 hour protein excretion of 12 grams
C. fundoscopic evidence of retinopathy
D. family history of diabetes

Show answer
Correct Answer: C

215. Which drug is the least likely to reduce proteinuria in a patient with
nephrotic syndrome?
A. high dose prednisone
B. ACE inhibitor
C. calcium channel blocker
D. angiotensin receptor blocker
E. cyclosporine

Show answer
Correct Answer: C

216. Thickening of the glomerular basement membrane is a late pathogenic event


in the development of membranous nephropathy.
A. True
B. False

Show answer
Correct Answer: A

217. Patients with nephrotic syndrome are at increased risk for developing
coronary artery disease.
A. True
B. False

Show answer
Correct Answer: A

218. A negative serum test for anti-neutrophil cytoplasmic antibodies (ANCA)


and a normal serum complement level exclude a diagnosis of vasculitis and
rapidly progressive glomerulonephritis.
A. True
B. False

Show answer
Correct Answer: B

219. The most likely diagnosis in an 18 year-old female with recurrent


episodes of macroscopic hematuria and persistent proteinuria is lupus
glomerulonephritis.
A. True
B. False

Show answer
Correct Answer: B

220. Which one of the following statements is correct?


A. An asymptomatic patient with a plasma sodium concentration of 154 mEq/L who
is hypotensive due to dehydration should be treated with 1/2 normal saline
instead of normal saline because of concerns over hypernatremia..
B. A patient with markedly altered mental status thought due to a plasma
sodium concentration of 105 mEq/L should be treated with 1/2 normal saline to
raise the plasma sodium concentration gradually.
C. A patient with markedly altered mental status thought due to a plasma
sodium concentration of 105 mEq/L should have the plasma sodium concentration
corrected to normal levels over the next 24 hours.
D. An asymptomatic patient with a plasma sodium concentration of 110 mEq/L
does not need emergent treatment and can be treated with water restriction.
E. A patient with markedly altered mental status and a plasma sodium
concentration of 165 mEq/L should be treated with IV D5W with the goal being
to normalize the plasma concentration over the next 12 hours.

Show answer
Correct Answer: D

221. The syndrome of inappropriate ADH secretion is usually characterized by


all of the following EXCEPT which one?
A. Low serum osmolality and hyponatremia
B. Sodium retention with low urine sodium concentration
C. Urine which is less than maximally dilute
D. Improvement in hyponatremia with water restriction
E. Presence of pulmonary disease, CNS disease, tumors or drugs

Show answer
Correct Answer: B

222. Which one of the following is FALSE with regards to diabetes insipidus?
A. Urine volume is typically large
B. Drugs are a common cause of nephrogenic diabetes insipidus
C. Central diabetes insipidus is commonly caused by trauma, surgery, or is
idiopathic
D. Plasma ADH concentration can be high in patients with nephrogenic diabetes
insipidus
E. Water restriction as a diagnostic maneuver generally can not distinguish
between central and nephrogenic DI

Show answer
Correct Answer: E

223. Which one of the following statements is correct with regards to the
physiology of renal water handling?
A. ADH release by the pituitary is primarily regulated by plasma osmolality
with relatively little contribution by plasma volume.
B. In order to make adequately concentrated urine it is not necessary to have
normal sodium reabsorption by the thick ascending limb.
C. In order to make adequate dilute urine the collecting duct must remain
impermeable to water.
D. ADH can increase urine osmolality by about 50-fold.

E. The thick ascending limb of Henle loop is permeable to water.

Show answer
Correct Answer: C

224. Edema formation in congestive heart failure is due to all of the


following EXCEPT which one:
A. Decreased cardiac output
B. Increased vasopressin release
C. Increased sympathetic nervous system activity
D. Portal hypertension
E. Increased activity of the renin-angiotensin-aldosterone system

Show answer
Correct Answer: D

225. Which one of the following is true about edema formation?


A. An intrinsic defect in renal sodium and water excretion is primarily
responsible for edema formation in the nephrotic syndrome.
B. Renal sodium retention in cirrhosis is not primarily due to splanchnic
vasodilatation.
C. Decreased hepatic albumin synthesis is the major cause of ascites formation
in cirrhosis.
D. Sodium retention in the nephrotic syndrome is almost always associated with
a marked reduction in GFR (<20 ml/min).
E. Hypoalbuminemia is the major factor in the initial sodium retention in the
nephrotic syndrome.

Show answer
Correct Answer: A

226. Which of the following is true about the treatment of edema?


A. The patient will not respond to diuretics if the fractional excretion of
sodium is less than 1%.
B. Intravenous fluids, even at maintenance rates, are rarely a cause of edema
in hospitalized patients.
C. Restricting dietary salt is the most effective way to reduce edema.

D. Diuretics are appropriate agents to give to edematous patients, however


their action may be limited if GFR is markedly reduced.
E. A fractional excretion of sodium greater than 2% is consistent with
congestive heart failure or cirrhosis.

Show answer
Correct Answer: D

227. Which statement is FALSE about the mechanism of edema formation?


A. Edema forming states are usually associated with reduced effective arterial
blood volume.
B. Edema forming states are always associated with increased actual blood
volume.
C. Increased proximal tubule salt and water reabsorption contributes to edema
formation
D. Enhanced distal nephron salt and water reabsorption contributes to edema
formation.
E. Renal blood flow is often reduced in edema-forming states.

Show answer
Correct Answer: B

228. A 40 year old male has diabetes for the past ten years. His blood
pressure is consistently about 140/90 mm Hg. He has 700 mg of protein on a 24
hour urine collection. You want to start him on antihypertensive therapy.
Which of the following would be the best choice in this situation?
A. Beta blocker
B. Calcium channel blocker
C. Thiazide diuretic
D. ACE inhibitors

Show answer
Correct Answer: D

229. Which of the following is not a risk factor for PRIMARY hypertension?
A. African American race
B. High salt intake

C. Renal artery stenosis


D. Obesity

Show answer
Correct Answer: C

230. You are treating a 55 year old obese man for his hypertension who is
tired most of the day. The most likely secondary cause for his hypertension
is:
A. Sleep apnea
B. Cushing syndrome
C. Pheochromocytoma
D. Renal disease

Show answer
Correct Answer: A

231. A low blood urea nitrogen to serum creatinine ratio (<10:1) is most
likely seen in which of the following conditions:
A. catabolic state resulting from sepsis
B. prerenal azotemia
C. high protein diet
D. rhabdomyolysis (muscle breakdown) from ischemia

Show answer
Correct Answer: D

232. For a given individual, an increase of serum creatinine from 5.0 mg/dl to
10.0 mg/dl represents a loss of fewer nephrons than an increase of serum
creatinine from 1.0 mg/dl to 2.0 mg/dl.
A. true
B. false

Show answer
Correct Answer: A

233. Which of the following is most likely to be associated with high urinary
sodium concentration (>20 mEq/L):
A. low salt diet in a normal subject
B. severe congestive heart failure
C. diuretic treatment for edema associated with chronic renal failure
D. renal vasoconstriction from non-steroidal anti-inflammatory agents

Show answer
Correct Answer: C

234. Which of the following is false regarding the fractional excretion of


sodium (FENa)
A. it requires the urinary sodium concentration for calculation
B. it requires the volume of the urine sample for calculation
C. it requires the urinary creatinine concentration for calculation
D. it is useful in distinguishing prerenal azotemia from acute tubular
necrosis

Show answer
Correct Answer: B

235. Which one of the following is not a recognized mechanism of acute renal
failure:
A. decreased renal blood flow as a result of tubulo-glomerular feedback
B. tubular obstruction from casts
C. back-leak of solutes as a result of tubular epithelial cell damage
D. mesangial cell relaxation

Show answer
Correct Answer: D

236. In a patient with oliguria, which one of the following is most consistent
with acute tubular necrosis rather than pre-renal azotemia:
A. urinary sodium concentration = 3 mEq/L
B. ratio of urine osmolality to plasma osmolality = 1.0

C. ratio of urine creatinine concentration to plasma creatinine concentration


= 80
D. fractional excretion of sodium (FENa) = 0.2%

Show answer
Correct Answer: B

237. In a patient with renal failure of uncertain duration and serum


creatinine of 8 mg/dl, which of the following tests is most useful in
assisting in the diagnosis of collecting system obstruction:
A. fractional excretion of sodium
B. ratio of blood urea nitrogen to serum creatinine concentration
C. renal ultrasound
D. intravenous pyelogram
E. renal angiogram with contrast

Show answer
Correct Answer: C

238. Which of the following is an indication for urgent dialysis in a patient


with renal failure:
A. blood urea nitrogen of 120 mg/dL
B. peripheral edema
C. serum potassium of 9.0 mEq/L
D. serum creatinine of 12 mg/dL
E. severe normochromic normocytic anemia with hemoglobin concentration of 7
g/dl.

Show answer
Correct Answer: C

239. Which of the following is false regarding the progression of chronic


renal failure:
A. angiotensin converting enzyme inhibitors and angiotensin receptor blockers
are often beneficial in retarding the progression of the kidney disease
B. kidney disease often progresses even after the initial insult (e.g.,
nonsteroidal anti-inflammatory agents) has been eliminated

C. blood pressure control does not influence the progression of renal disease
D. adjustment of medications is often necessary in order to avoid systemic
toxicity of the medications

Show answer
Correct Answer: C

240. The most common cause of end stage renal disease in the U.S. is:
A. post-infectious glomerulonephritis
B. polycystic kidney disease
C. toxic nephropathy from antibiotics
D. sports injury of kidneys
E. diabetic nephropathy

Show answer
Correct Answer: E

241. Which of the following is not involved in the pathogenesis of osteitis


fibrosa cystica:
A. elevated serum parathyroid hormone level
B. hyperphosphatemia
C. accumulation of beta-2-microglobulin
D. decrease in serum 1,25-dihydroxy-vitamin D3 levels predisposing to
hyperparathyroidism

Show answer
Correct Answer: C

242. Which one of the following is not a typical systemic complication of


chronic renal failure:
A. anemia
B. insulin resistance in patients not known to have diabetes
C. left ventricular hypertrophy
D. decreased serum triglyceride levels
E. neuropathy

Show answer
Correct Answer: D

243. Which is not true of childhood hemolytic uremic syndrome


A. There is usually a diarrheal prodrome
B. It is described by acute renal injury, microangiopathic hemolytic anemia,
and thrombocytopenia
C. Recovery of renal function rarely occurs
D. Neurologic involvement is common

Show answer
Correct Answer: C

244. The most common cause of bladder outlet obstruction in boys is


A. Eagle-Barrett Syndrome (Prune Belly Syndrome)
B. Autosomal recessive polycystic kidney disease
C. Multicystic dysplasia
D. Posterior urethral valves

Show answer
Correct Answer: D

245. Fractional excretion of sodium (FENa) in newborns


A. Is lower in infants than older children and adults
B. Contributes to the negative sodium balance observed in premature infants
C. Is unrelated to gestational age at birth
D. Has never been measured

Show answer
Correct Answer: B

246. Renal dysplasia


A. Is the result of abnormal mesonephric development
B. Is influenced by abnormal inductive effects of the ureteric bud on the
metanephros

C. Describes only one developmental disorder


D. Does not involve primitive elements in the kidney (e.g. cartilage,
primitive tubules)

Show answer
Correct Answer: B

You might also like